× Register Login What's New! Contact us
Page 1 of 7 1 2 3 ... Last
Results 1 to 20 of 124 visibility 496232

The Medical student Review

  1. #1
    جوري's Avatar Full Member
    brightness_1
    Soldier Through It!
    star_rate star_rate star_rate star_rate star_rate star_rate star_rate star_rate star_rate star_rate star_rate
    Join Date
    Jul 2006
    Location
    من ارض الكنانة
    Gender
    Female
    Religion
    Islam
    Posts
    27,759
    Threads
    1260
    Rep Power
    259
    Rep Ratio
    89
    Likes Ratio
    23

    The Medical student Review

    Report bad ads?

    I don't know how many med students there are on board, or how useful this review will be for you.. I am a firm believer in slow and consistent than an intellectual enema two days before the exam where once released never again to be regained.. So what I plan to do here, is share some things I feel are important..

    In my pre-clinical years, I was president of the pharmacology club, and I enjoyed teaching it and exchanging ideas with others.. so if you have your own forte, quirks mnemonics that you'd like to share we can make a useful compendium..

    I am only going to focus here on pharmacology and diagnostic testings.. so every day I'll give you five of each..

    I am not going to start in any particular order but once in a section, I plan to complete it.. this will be just the high yield..


    __________________________________________________ ___________

    Oncology diagnostic testing
    _____________
    AFP (what disease is associated with it, and when do you answer this for a question?)
    AFP is associated with the development of
    1-hepatocellular ca.
    2-ovarian cancer
    3-non-seminomatous germ cell tumors
    Answer AFP when you see a patient with alcoholic cirrhosis or chronic hepatitis B or V. AFP together with radiological imaging is used to screen for hepatocellular ca

    ____________________________________________
    CEA
    1-A protein elevated in a variety of cancers including colorectal cancer
    2-serum CEA have a prognostic value in pts with newely diagnosed CRC. Those with higher levels have worse prognosis.
    3-CEA level to monitor in colon cancer in pts after a surgical resection, it determines the presence of persistent, recurrent or metastatic disease

    __________________________________________________ ___________
    Colposcopy
    Colposcopy is the direct visualization of the cervix, by use of a magnifying scope with a lamp
    2-The transition zone must be visualized to ensure an adequate colposcopy. The border between squamous and columnar epithelium
    3-a colposcopy is the answer for a pt with an abnromal pap
    Atypical squams can't exclude high grade lesion ASC-II
    low grade squamous intraepithelial lesions LSIL
    high grade intraepithelial lesions HSIL
    Atypical sqams of undetermined significance ASCUS if HPV DNA testing is positive
    __________________________________________________ ___________
    Estrogen and progesterone receptors
    Should be done on all pts with breast cancer, in order to determine who should receive hormone therapy
    2-Therapy with either tamoxifen or raloxifene should be added to any pt. with positive receptors. This is either for estrogen or progesterone positivity alone or in combination. The response to tamoxifen is better if both receptors are positive
    __________________________________________________ ____
    Mammogram
    screening should begin at age 40 and should be performed every 1~2 years, screening at age 50 should be yearly
    2-when mammogram shows abnormalities, a core biopsy including sentinel lymph node biopsy is the next best step. carcinomas of the breast are associated with clustered polymorphic microcalcifications.
    3-screening lowers mortality most after age 50 and the dec is greater than that of a colonscopy or a pap smear
    __________________________________________________ ________________

    Now pharm
    _____________

    will start with infectious disease because it is the longest chunk
    Acyclovir/valcyclovir/famiciclovir

    all the above are the correct answer for
    Herpes simplex including, genital, cutaneous, orolabial
    for Herpes Encephalitis (acyclovir) IV form only in a hospital setting
    Varicella zoster
    shingles: Herpes zoster or reactivation
    Bell's palsy

    the above meds work by inhibition of thymidine kinase
    -most common adverse effects are nephrotoxicity presumably from precipitation of the meds in the kidney tubule, sx of neurological toxicity in the kidney tubule, sx of neurological tox, such as confusion, tremors and hallucination occur rarely
    __________________________________________________ __
    Rifaximin
    used to treat travelers' diarrhea such as that from E.coli, it isn't used for invasive diarrhea. an associated fever and bloody diarrhea indicate and invasive pathogen, such as campylobacter. When fever and bloody diarrhea are described, ciprofloxacin is the best answer.

    rifa is a nonabsorbed version of the rifamycin antibiotic, it inhibits ribosomal RNA production of essential proteins
    There are no major side effects since it isn't absorbed from the GI tract. it doesn't cause C.Diff, and may in fact treat it.
    _________________________________________________
    Daptomycin and Linezolid

    they are both used for gram +ve organisms such as MRSA, streptococci and vanc resistant enterococci. Linezolid is the only oral antibiotic for MRSA. They can both be used for Vanc resistant organisms .

    Liezolid is an oxazolidinone and inhibits protein synthesis at the ribosome. Daptomycin is a cyclic lipopeptide and disrupts cell membrane they are both unique classes of drugs

    Linezolid commonly causes thrombocytopenia, and is a MAO inhibitor, avoid tyramine foods. Dapto caused CPK on liver functiion tests to be elevated!!
    __________________________________________________ _____________
    Tigecycline
    an extremely broad spectrum anti-biotic that covrers MRSA, staph aureus, and well as gram negative bacilli. Tigecycline is the answer for complicated hospital of ICU acquired infections, tigecycline alone is equivalent to vanc and aztreonam in combination, it is also active against resistant enterococci and PCN resistant penumococcus

    tigecycline is a glyclycyline antibiotic that binds to the ribosome and inhibits protein synthesis, it is unique that it covers staph, strep, gram negatives, anerobes and organisms resistant to vanc
    tigecycline is hepatotoxic , caused nausea and diarrhea
    __________________________________________________ ________

    Polymyxin B (PMB) and colistin
    useful for conjunctivitis, infections of the skin, and otitis externa, also correct for multi drug resistant gram negative bacilli, that cause ventilator associated pneumonia and sepsis from pseudomonas or acinetobacter.

    works by disrupting phospholipids in the cell wall membrane
    Polymyxin B (PMB) and colistin are very toxic to the kidney and nerves and is limited to topical applications of the skin and ear, they are also used for multi drug resistant gram negative bacilli when there are no other therapeutic options..

    __________________________________________________ ______
    That is it for today..
    The Medical student Review

    Text without context is pretext
    If your opponent is of choleric temperament, seek to irritate him 44845203 1 - The Medical student Review


  2. Report bad ads?
  3. #2
    جوري's Avatar Full Member
    brightness_1
    Soldier Through It!
    star_rate star_rate star_rate star_rate star_rate star_rate star_rate star_rate star_rate star_rate star_rate
    Join Date
    Jul 2006
    Location
    من ارض الكنانة
    Gender
    Female
    Religion
    Islam
    Posts
    27,759
    Threads
    1260
    Rep Power
    259
    Rep Ratio
    89
    Likes Ratio
    23

    Re: The Medical student Review

    ok today's 5 and 5

    in diagnostic testing, I am finishing the last two in oncology and starting three in cardiology, and we'll continue cardio tomorrow insha'Allah

    Diagnostic onco/cardio

    PET scanning
    ________________
    Positron emission tomograpy
    this measures the metabolic activity of a lesion seen on a CT or an MRI scan. It is based on the ability of most cancers and some infections to have an increased utake of 18-flurodeoxyglucose. it is a noninvasive test of the actual function of a mass lesion, not just its size and location.

    most cancers and some infections have an increaed iptake of tagged glucose

    A pet sca can tell if a lesion, such as a lung mass, that might otherwise look malignant, really is maliganant, for instance, if you are show a case of a localized lung ca. diagnosed with a biopsy, you can use the PET scan to see if there are metastases. if there s a mass in the contalteral chest that is malignant, it makes the pts. ineligible for surgery with curative intent. If the PET scan shows a low uptake of the tagged glucose, then the lesion is likely benign, and you should proceed with the surgery to remove the promary tumor site.
    _______________________________________
    Sentinel lymph node eval
    it is used in the eval of breast cancer
    the sentinel node biopsy is when dye is introduced into the operative field, the first node it goes into is the sentintel node. if it has cancer, you dissect the axilla. if it is negative, you don't do an axillary lymph nde dissection.

    look for a pt with abnormal mammogram in whom the biopsy shows cancer, then the answer is sentintel node biopsy. Sentinel node biopdy is the best test to do after the initial diagnsis of breast cancer has been made with either excisional or needle biopsy. sentinel node biopsy can eliminate the need for axillary lymph node dissection.
    __________________________________________________
    PSA
    increases can occur in prostatitis, BPH and prostate cancer, levels are undectectable after a total resection of the prostate.

    PSA testing is extremely conttoversial. There is no evidence that PSA testing lowers mortality when used as a routine screening test.
    Biopsy of the prostate is most accurate test.
    If the PSA is elevated, a DRE is performed to palpate a lesion. if a Lesion is found, it should be biopsied, if no lesion is found, a transrectal ultrasound should be performed to find a lesion to biopsy. If no lesion can be found, then 'blind biopsies' implemented

    Ok on to cardio diagnostics

    case 32 s 2 1 - The Medical student Review

    easy one EKG shows asystole,
    there is no activity of any kind, astystole will not however give a perfect flat line. there will still be modest baseline undulation.

    Asystome occurs in a pt who suddenly loses pulse and BP. but other causes of pulselessness have been excluded. It is not possible to distinguish between asystole, ventricular fibrillation, pulseless electrical activity from tamponade, or tension pneumothorax without an EKG.

    Atrial20Fibrillation20in20the20Dog1 - The Medical student Review

    this EKG shows atrial fibrillation (Afib)
    it is charcterized by a normal QRS duration of <120 miliseconds and the presence of fibrillatory waves. Atrial fibrillatory waves eliminate any organized atrial activity such as a P-wave or even a flutter wave. The atrium is having a 'seizure' the rhythm is irregulary irregular. There is no fixed pattern between the R waves.

    Afib pts presents with palpitations and has an irregularly irregular heart rhythm. the HX will often contain CHF, HTN, or hyperthyroidism. although simple caffeine excess is also a possibility.
    __________________________________________________ _________
    ok now
    Pharm
    still on with infectious disease

    what do you fo for someone who has been bitten by a dog, and a dog that is has been bitten by a human being?

    bites are treated with local wound care and tetanus prophylaxix, antibiotics aren't routinely indicated.
    do answer antibiotics in the following
    hand bite wounds
    deep puncture wound, wounds requiring surgical debridement, older pts, bites near a prosthetic joint or any infected wound, if antibiotics are indicated.
    Amoxicillin/clavulanic acid is the best antibiotic to use and PCN allergic pts are given clindamycin and trimethoprim/sulfamethoxazole.
    Human bites transmit Eikenella corrodens, DOgs and cats transmit pasturella multocida

    two cases, a 28 yr old from massachusetts landing from New Zealand, the evning after she arrives she finds a tick attached to her ankle, which treatment should she use?
    simple reassurance, this is an asymptomatic tick bite from a country that is not endemic. in addition the tick has been attached for <24 hrs. Ticks, even if infected need more than 24, 36 hrs to transmit the organism for lyme dz.

    2nd scenario
    7 year old goes camping in the woods in connecticut, a weej after, he devoped a circular erythematous rash with central clering that is >5 cm what is the best RX
    amoxil
    there is no need for serology since the rash is characteristic. Lyme rash is typically >5cm. doxycycline and amoxil are equally efficatious, but there is no need to use doxy on a child less than 8 yrs old.
    __________________________________________
    23 yer old comes in with myalgia, fever, cough, headache, arthralgia, and sore throat for 24-36 hrs

    best therapy is oseltaivir and zanamivir, both neuraminidase inhibitors that can be used to RX influenza A and B, within 48 hrs of the development of sx. They shorten the duration of illness 1-2 days.
    the inhibition of neuraminidase prevents the realease of virus from cell surfae
    zanamivir may cause wheezing because it is administered by inhalation
    ______________________________
    Man with AIDS comes into ER with blurry vision, he is found to have a retinal lesion and his CD4 count is <50/MM3

    CMV retinitis is the most common cause of retinal lesion in pts with AIDS and low CD4 counts, the best therapies are Valganciclovir, ganciclovir, foscarnet or cidofovir, because it can be sed orally valganciclovir is the drug of choice.
    all these drugs interefere with the viral replication of CMV
    the adverse effects are
    Ganciclovir:neurtopenia
    Foscarnet: nephrotoxicity, urethral ulcers, hypocalcemia, hypomagnesemia.
    cidofovir: nephrotoxicity
    valganciclovir: seizures, neutropenia
    ________________________________________________

    drotrecogin the famous XIGRIS
    used for severe sepsis with multiorgan dysfunction and a high APACHE II score,
    it reduces mortality and sepsis and is used in addition to antibiotics, fluids, possible pressorts such as norepi. if a pt with hypotension and tachycardia, hyperventillarion and hypoxia from sepsis leading to ventillator dependence, then drotrecogin is indicated.
    drotrcogin is an activated protein C analog. it has fibrinolytic and anti-inflammatory properties
    the most common adverse effect is bleeding..
    The Medical student Review

    Text without context is pretext
    If your opponent is of choleric temperament, seek to irritate him 44845203 1 - The Medical student Review


  4. #3
    جوري's Avatar Full Member
    brightness_1
    Soldier Through It!
    star_rate star_rate star_rate star_rate star_rate star_rate star_rate star_rate star_rate star_rate star_rate
    Join Date
    Jul 2006
    Location
    من ارض الكنانة
    Gender
    Female
    Religion
    Islam
    Posts
    27,759
    Threads
    1260
    Rep Power
    259
    Rep Ratio
    89
    Likes Ratio
    23

    Re: The Medical student Review

    Today's 5/5
    diagnostics still cardio

    Flutter 1 - The Medical student Review
    This is an atrial flutter, the QRS is of normal duratio <120 MSEC and there is a saw tooth pattern to the atrial waves. The R-R intervals are regular. The flutter waves are identical to each other. This regularity as well as the presence of atrial activity is what distinguishes this from narrow complex tachycardia and Afib

    the patient will have palpitations and maybe lightheaded. the pulse will be regular. The rate is often slower that that of SVT. You can't distinguish atral flutter from SVT without an EKG they both have the same heart rate.
    ____________________________________________
    supraventricular tach 1 - The Medical student Review

    This is supraventricular tachycardia (SVT). It is a regular tachycardia with narrow QRS complexes. and no apparent P waves, fibrillatory waves or flutter waves. A normal QRS duration <120 MSec

    the pt will have rapid palpitations and lightheadedness, triggers include caffeine or other sympathetic system stimulating drugs.. psychological stress and hyperthyroidism .
    RX includes a carotid message, followed by IV adenosine. If adenosine doesn't work, then a calcium channel blocker, e.x verapamil or diltiazem, digoxin or beta blockers can also be tried

    __________________________________________________ _
    1013900401gr4 - The Medical student Review
    multifocal atrial tachycardia
    Mat is characterized by irregularly irregular rhythm as demonstrated by variability in the P-P intervals. There are multiple p-R intervals and at least three different P wave morphologies. The atrial rate is over 100 per minute, when the rate is lower it is no longer tachycardia it is now called 'wondering pacemaker'

    MAT is caused by right heart strain and the effect of chronic pulmonary disease on the right side such as in COPD
    -__________________________________________________
    heart 02 - The Medical student Review

    Torsades de Pointes

    The most distinctive feature is the undulating amplitude that gives the impression the EKG is twisting around a point. Torsdes is a form of wide complex tachycardia. The QRS width or duration is >120 MSEC. essentially, Torsades is ventricular tachycardia with an undulating amplitude.

    Just like ventricular tachy, Torsades may present with anything from simple palpitations to syncope to sudden death. Can't tell without the EKG. Torsades can also be caused by toxicity to meds such as tricyclic antidepressants, amiodarone, dofetilide, ibutilide, macrolides and rarely quinolone antibiotics. . Look for hypomagnesemia.
    ________________________________________________
    vfib 1 - The Medical student Review

    this is Ventricular fibrillation
    The EKG has no recognized activity at all, there maybe either high or low amplitude to the EKG. if you see organized P waves, QRA, or regular rhythm then it isn't VFIB. But there is electrical activity so it isn't a flat line like asystole
    Pts with Vfib, have no pulse, no respirations. They have no blood pressure, and have suffered a cardiac arrest. you can't distinguish between vfib, pulseless electrical activity and certain forms of pulseless ventricular tachycardia without an EKG

    __________________________________________________ ______

    Now Pharm, continuing on with infectious DZ.
    An intern gets stuck by a needle from a known AIDS pt.
    HIV infected injuries should be treated with the following prophylactic regimens
    Zidovudine, lamivudine and a protease inhibitor (or efavirenz)
    or
    Tenofovir, emtricitabine and a protease inhibitor or (efavirenz)
    the recommended protease inhibitor is lopinavir/ritonavir.
    the alternatives are atazanavir, fosamprenavir or nelfinavir.
    The risk of transmission is 0.2% or 1 in 300 without therapy
    Nausea and vomiting are the most common adverse effects of therapy.
    __________________________________________________ __

    a middle aged man going for a visit to india

    should be given mefloquine or atovoquone/proguanil for malaria prophylaxix. Doxycycline is used when there is mefloquine resistance. chloroquine can be used in those areas in which the malaria is still sensitive to it. chloroquine sensitivity is retained in central america and some parts of the middle east.
    Mefloquine is associated with neuropsychiatric abnormalities, such as psychosis, hallucinations and seizures. It may cause cardia rhythm effects. such as bradycardia, and QT interval prolongation. if the person is on beta blocker or having an arrhythmia, use atovoquone/proguanil instead of mefloquine.
    Doxycycline is asscoaited ith photo sensitivity.
    _______________________________________________

    nurse struck by a needle from a hep B surface antigen Positive pt.
    and a middle aged trauma surgeon has been lacerated by a scalpel with blood from a pt with hep C
    approach to both.
    case one, check her Hep B status. if she has protective levels of antibody, then no further post-exposure prophylaxis is needed. The presence of surface antibodies indicated that the person is protected. if the injured has no protective antibody, then she should receive hep B immune globulin and a full course of hep B vaccine

    case 2. there is nothing you can do. there is no prophylaxis for hep C
    __________________________________________________
    child wakes up in the middle of the night to find a bat in his room, which flies out, no injury is noted..
    case 2. Man finds his neighbor's dog in his garden, when he chases him the dog bites him..

    case one, the most common vector for transmission of rabies in the U.S are bats. prophylaxis is indicated with rabie immune globulin and rabies vaccine for any level of injury, including bites, scratches and mucus membrane exposure. prophylaxis is also indicated if the bat is found in a room while someone is sleeping, and contact is undetermined.

    case 2, rabis prophylaxis doesn't need to be given for dog bites, if the dog is known as healthy. if the dog is wild or unknown, then the dog should be observed for ten days. if abnormal behavior develops, then prophylaxis should be offered. Pt with bites from a dog with abnormal behavior at any time should receive rabies immune globulin and vaccine.
    ____________________________________________

    Woman working in a construction site and is accidentally hit by a nail, that was lying in the dirt, she received no vaccines since childhood

    determine RX as follows.
    prophylaxis for tetanus, should be given with any wound if more than ten yrs have passed since the last vaccination. if a vaccination has been given previously, prophylaxis consists of only tetanus toxoid.
    If a person has never been vaccinated, then tetanus toxoid and tetanus immune globulin should be given. only people who have never been vaccinated should be given immunoglobulin.
    If the person has received a dirty wound , then a booster of tetanus toxoid should be given if more than five years have passed since the last booster. clean .
    clean wounds are protected for twn yrs.. dirty wounds for five years.
    The Medical student Review

    Text without context is pretext
    If your opponent is of choleric temperament, seek to irritate him 44845203 1 - The Medical student Review


  5. #4
    جوري's Avatar Full Member
    brightness_1
    Soldier Through It!
    star_rate star_rate star_rate star_rate star_rate star_rate star_rate star_rate star_rate star_rate star_rate
    Join Date
    Jul 2006
    Location
    من ارض الكنانة
    Gender
    Female
    Religion
    Islam
    Posts
    27,759
    Threads
    1260
    Rep Power
    259
    Rep Ratio
    89
    Likes Ratio
    23

    Re: The Medical student Review

    Today's 5 and 5 still in cardio and infectious respectively

    ventricular tach 1 - The Medical student Review

    This is wide complex tachycardia (ventricular tachycardia)
    the pt. presents with syncope, lightheadedness, sx of CHF or sudden death, the range is broad and there maybe palpitations

    The rx depends on the pts stability
    for unstable pts we use cardioversion
    stable pts amiodarone, lidocaine, or procainamide
    ______________________________________________
    wolffparkinsonwhite 1 - The Medical student Review
    Wolff-Parkinson-White Syndrome
    this is a ventricular pre-excitation syndrome
    The EKG shows a short PR and a slurring of upstroke of the QRS known as delta waves
    the pt ca have supraventricular tachycardia alternating with ventricular tachycardia. There will be pre-excitation on EKG and paroxysmal tachycardia. The SVT maybe worsened after ca2+ blocker or digoxin
    this is an accessory pathway for cardiac conduction from the atria to the ventricles that bypasses the AV node and causes earlier activation (pre-excitation) of the ventricles.
    __________________________________________________ _____

    Coronary angiography or cardiac cath
    placement of a catheter from the brachial or femoral artery to the coronary artery with injection of iodinated contrast
    angiography is the answer when
    the cardiac stress test is abnormal
    angiography provides an exact measure of degree of stenosis of the coronary artery and the number of vessels involved, The degree of stenosis needs to be >70% to be considered significant
    __________________________________________________ ____
    BNP
    a hormone released from the heart and the brain in response to stretch in the atria and ventricles. BNP has diuretic, natriuretic and hypotensive effects and inhibits the renin angiotensin system in response to fluid overload states.
    BNP goes up with CHF and other causes of hypervolemia

    and elevated BNP is confirmed by an echo
    __________________________________________________ ____

    Pharm/ ID

    Flu Vaccine
    Everyone over 50
    health care workers
    anyone with COPD, liver cirrhosis, or kidney dialysis, diabetes, HIV, or who uses steriods, has no spleen, ir is a resident of a chronic health care facility
    pregnant women
    Flu vaccine is constructed with surface viral antigens, neuroaminidases, hemagglutinins from the previous year's most important strains
    flu vaccines are given yearely
    the vaccination can induce a mild fever, muscle soreness, very rarely associated with Guillain-Barre syndrome.
    Influenza vaccine is contraindicated when the pt is allergic to eggs. the live attenuated intranasal vaccne should not be given to pregnant or immunocompromised pts.
    __________________________________________________ ____
    pneumococcal vaccination
    should be administered to
    everyone over65
    anyone with COPD, liver Cirrhosis, kidney dialysis, diabetes, or HIV or who has no spleen, uses steriods, or resides in chronic care facility
    alcoholics
    people with cochlear implants
    this vaccine is derived from the capsular polysaccharide surface of the pnumococcus
    most ts over 65 receive a single injection, a second injection is give after five years only in immunocompromised pts and those in whom the first injection was before age 65
    penmococcal vaccine is safe
    _______________________________________________

    clindamycin
    best therapy for predmonantely anerobic
    such as lung absess
    aspiration penumonia from oral or abdominal anerobes
    it is also useful in treating gram positive organisms, such as staphlococci and step. although not as effective as a beta lactam. like PCN or cephalosporin. It is used when pts has a life threatening anaphylactic reation to PCN
    ex of such infections
    include
    cellulitis, erysipelas and impetigo
    oral infections
    pelvic infections
    other second line uses include rx for the following
    Penumocystis pneumonia
    bacterial vaginosis
    toxoplasmosis
    MRSA
    clindamycin is licosamide atntibiotic it inhibits ribosome, it often leads to dirrhea and c.diff
    __________________________________________________ _
    Metronidazole

    effective against anerobic gram negative bacilli found in the abdomen, example of treatable conditions include
    C.diff colitis
    diverticulitis in combo with quinolone
    perforated abdminal absess
    pelvic infections, PID, Vaginitis
    it is also effective agsinst potozoal infections
    such as Giardia
    enamoeba histolytica
    trichomonas
    Metronidazole inhibits bacterial mitochondia, interferes with numerous cystosolic products, also blocks
    ____________________________________________

    1- pt comes in for follow up care for TB. complains of blurry vision, he is on isoniazid, rifampin, pyriziniamide, and ethambutol
    shows no sx that he has developed hypruricemia after six weeks of TB meds

    stop the ethambutal, it causes optic neuritis and can cause bluryr vision, color vision abnormalities can be the first sx.
    hyperuricemia is most likely from pyrizinamide, however because the pt is asymptomatic, you should not stop meds. pyrizinamide is essntial to shorten the duration of therapy from 9 months to six months

    isonizid causes peripheral neuropathy, and its use should be accompanices by pyridoxine replacement. Rufampin causes red orange discolaration of body fluids but does not need to be stopped.
    All TB therapy can result in hepatotoxicity..
    The Medical student Review

    Text without context is pretext
    If your opponent is of choleric temperament, seek to irritate him 44845203 1 - The Medical student Review


  6. Report bad ads?
  7. #5
    جوري's Avatar Full Member
    brightness_1
    Soldier Through It!
    star_rate star_rate star_rate star_rate star_rate star_rate star_rate star_rate star_rate star_rate star_rate
    Join Date
    Jul 2006
    Location
    من ارض الكنانة
    Gender
    Female
    Religion
    Islam
    Posts
    27,759
    Threads
    1260
    Rep Power
    259
    Rep Ratio
    89
    Likes Ratio
    23

    Re: The Medical student Review

    Today's 5/5

    starting wtih pharm still on infectious dz
    _______________________
    42 year old homeless, immigrant, health care worker, alcoholic presents with fever, cough, sputum, and weight los over tthree months. chest x-ray shows apical infiltrate, and sputum is + for acid fast bacilli

    TB cases should be placed on respiratory isolation and started on foru TB meds. a positive smear is sufficient to initiate therapy. INH, rifampin, pyrizinamide and ethambutal are medications for initial therapy.

    All four meds should be used for the first two months, after sensitivities are known, ethambutal an pyrizinamide can be stopped. INH and rifampin should contiunue for four more onths for a total of 6 months therapy

    standrd rx is 6 months therapy. RX is prolonged is the pt has the following osteomyelitis, meningitis, pregnancy, or milliary TB
    __________________________________________________ ___
    25 year old medical student is entering his residency. his PPD has a 12MM of induration. He had Bacille Calmette-Guerin (BCG) as a child, he has no sx, and his chest x-ray is normal

    the student has a PP with a >10MM of induration and a normal x ray, he should receive 9 months of isonizd. the hx of BCG is not important, you can't distinguish a true + from TB exposure in the past from a false + from BCG vaccination
    a + PPD confers a 10% lifetime risk of TB. Most reactivation occurs within the first two yrs of exposure
    Isonizid reduces this 10% risk by 80-90%
    isonizid can cause hepatotoxicity. routine monitoring of LFT's is not necessary unless underlying liver dz or alcoholism is present.
    __________________________________________________ __

    Posaconazole

    is used as an empric theray with fever and neutropenia in pts who have no responded to empiric RX antibacterial coverage. Accepted empiric coverage is with imipienem or meropenem, cefepime or piperacillin/tazobactam. It isn't known whether posaconazole is superior or the echiocandins or voriconazole for febrile neutropenia. Posacaonazole is effective in the following conditions
    candida
    aspergillus
    mucormycosis
    cryptocococcus, histoplasmosis, coccidiodiomycosis

    azole antifungals block ergosterol synthesis
    adverse is liver tox. or QT prolongation may also occur.
    __________________________________________________ _______________
    echinocandins
    Caspofungin
    micafungin
    anidulafungin

    echinocandins such as caspofungin, are the best initial therapy in the rx of pts with fever and neutropenia who are still febrile after using bacterial antibiotics. echinocandins are at least as efficacious as amphotericin and much less toxic. Echinocandins are used to rx the following conditions
    candida infxns
    esophageal candidiasis
    aspergillus especially if other therapy fails
    echinocandins don't affect cryptococcus
    echinocandins blocks 1,3-D-Glucan. this is a polysaccaride in the fungal wall that doesn't exist in humans.
    these meds have few adverse rxns
    minor histamine release, flushing, headache, utracaria, and pruritus.
    __________________________________________________ ___________

    a pt with acute leukemia has recently undergone allogenic bone marrow transplantation, after two weeks of fever and neutropenia, she develops a lung infection diagnosed as aspergillus
    RX with voriconazole which is superior to amphotericin in the management of aspergillus. voriconazole results in greater survval with less adverse effects than amphotericin. itraconazole is not powerful enough in severe cases of aspergillus. Echinocandins (caspofungin) are used as a salavage if the pt. fails to respond to amphotercin or voriconazole. voriconazole will not treat mucormycosis.
    imidazoles and triazoles inhibit microsomal cytochrome P450 (CYP) enzymes. this impairs biosynthesis of ergosterol for the cytoplasmic membrane. amphotericin is a polyene antifungal that interfers with sterol production in the cell membrane .
    amphotericin will raise the creatining level and cause distal renal tubular acidosis with hypokalemia. voriconazole can be hepatotoxic like all azoles, it can prolong the QT on an EKG. the most unique adverse effect of voriconazole is a transient visual disturbance.
    __________________________________________________ ____
    6 diagnostics still in cardio I added an extra one because I know how excited you are to read these?
    CK-MB

    this is elevated serum marr for Myocardial injury. CK-MB is useful in detecting infarction and therefore the right answer in assessing any pt with chest pain especially with cardiac factors. CK-MB normally increases within 4-6 hrs fom the start of chest pain, it peaks about 12-24 hrs. C-MB is particularly useful in detecting re-infarction.

    The EKG doesn't have to have St segment elevation in order for Ck-MB to be abnromal. also the answer whrn chest pain re-occurs after an infarction within the last few days
    ______________________________________
    Dobutamine stress

    stress testing is a form of exercise intolerance or stress testing that does not use physical exercise. it is a noninvasive method to determine myocardial perfusion
    Dobutamine is a direct acting agonist at beta-1-adrenergic receptors, causing inotropic stress. dobutamine is injected and the echo is observed for a decrease in the wall motion. ischemic myocardium does not move as well as normal myocardium.

    dobutamine is used in pts who can't exercise look for a person who has a hx of chest pain but in whom the story is equivcal and and the EKG is nondiagnostic. The indications are the same as those for persantine stress testing. it is also used for pts with reactive airways diseases who can't undergo a persantine thallium test

    the coronoary angiogram is the most accurate test of myocardial perfusion.
    __________________________________________________ ________
    Cardiac electrophysiological EP studies
    and EP study is an introduction of a catheter with an electrical sensing and stimulating electrode into the heart. This allows both the detection of abnromal cardiac rhythm disturbances as well the stimulation of the heart to determine the sire of an abnromal rhythm.

    EP studies are used for
    defintive diagnosis of abnromal cardiac rhythm, particularly the etiology of unexplained syncope.
    mapping of the cardiac conduction system
    ablation of abberant conduction tract abnromalities such as SVT or wolff-parkinoson-white syndrome
    placement of automatic implanted cardioverter/defbrillator
    searching for the site of origin of sustained ventricular tachycardia.
    _________________________________________
    Ergonovine test
    involves the injection of ergonovine to provoke coronary vasospasm. this is diagnostic to detect prinzmetal's angina which is coronary vasospasm.
    it is the most accurate test when they give a younger pt <45 complaining of atypical anginal type chest pain who has clean cornories on cardiac cath. the pain will not bear a fixed reltionship to exercise
    prinzemetal variant angina is treated with a ca2+ chanel blocker.

    __________________________________________________ _
    Holter monitoring or 24 hr continuous ambulatory cardiac monitoring

    is a way of recording an EKG for 24 hrs of continuous reading from a single lead
    the pt wears leads on his chest and carried a recording device around his neck like a portable CD while at home. the strop can then be analyzed at high speed by the physician to detect a brief rhythm disturbance that may not have been detected on the original EKG
    abnromalities on the holter can be further analyzed by electrophysiological studies in a labortatory by echocardiogram.

    answer holter monitor when a pts has palpitation or syncope as the ekg does not detect an abnormality.
    _______________________________________________
    MUGA scans (Multiple Gated Acquisition scan)

    this is the single most accurate method for assessing ejection fraction. MUGA also decreases left ventricular wall motion and cardiac muscle damage

    MUGA is performed by injecting RBC radiolabeled with technetium 99 into the ps' blood stream and recording the emissions with a gamma camera.
    MUGA is very useful in assessing and following cardiac function in pts during the delivery of potentially cardiotoxic cemotherapy such as adrimycin. the accuracy and reproducability of the study make it suitable to detect subtle, early changes in cardiac function that might easily be missed by other techniques such as an echo....
    The Medical student Review

    Text without context is pretext
    If your opponent is of choleric temperament, seek to irritate him 44845203 1 - The Medical student Review


  8. #6
    جوري's Avatar Full Member
    brightness_1
    Soldier Through It!
    star_rate star_rate star_rate star_rate star_rate star_rate star_rate star_rate star_rate star_rate star_rate
    Join Date
    Jul 2006
    Location
    من ارض الكنانة
    Gender
    Female
    Religion
    Islam
    Posts
    27,759
    Threads
    1260
    Rep Power
    259
    Rep Ratio
    89
    Likes Ratio
    23

    Re: The Medical student Review

    Today's 5/5
    still on with infectious disease in phatm

    50 year old man with AIDS comes to the ER dept with S.O.B, dry cough, increased LDH, a pO2 of 64 and CXRAY with bilateral interstitial infiltrate. He has a CD4 count 105 and is on no medications:

    Pneumocystis pneumonia PCP is initially treated with IV TMP/SMX, milder cases of PCP without severe hypoxia can be treated with atovoquone
    adverse effect of TMP/SMX is a rash, bone marrow suppression, secondary to its ability to inhibit folate metabolism and renal insufficiency can also occur.

    if the pt is allergic to sulfa meds, pentamidine is the alternative for severe PCP

    sterioids are added for severe PCP, which is defines as pO2 <70 or an Aa gradient >35 (know how to calculate Aa gradient )PAO2 = ( FiO2 * (760 - 47)) - (PaCO2 / 0.8)
    therefore A-a gradient = PAO2 - PaO2

    that is arterial - alveolar but I digress
    you should know also that Normal A-a gradient = (Age+10) / 4

    __________________________________________________ ______

    24 year old man comes to clinic with urethral discharge/ pregnant woman with cervicitis

    urethritis is treated for both chlamydia as well as gonorrhea.. for chamydia we use a single dose azithromycin or doxycycline for a week. for gonorrhea use a single dose cefixime or ceftriaxone. Quinolones such as a single dose ciproflixicin, levofloxacin or oflaxicin can be used for gonorrhea if the rate of resistance is the community is low.

    cervicitis is managed the same way as urethritis, pregnany women however, should not use doxycycline or quinlones in pregnancy, use azithromyin and ceftriaxone.
    __________________________________________________ ______

    woman comes to the ER with lower abdominal pain and tenderness, fever, and cervical motion tenderness. pregnancy test is negative

    outpatient therapy for PID consists of levofloxacin alone for two weeks or ofloxacin and metronidazole. an alternate outpatient therapy is ceftriaxone or cefoxitin with doxycycline for two weeks.
    inpatient therapy for PID consists of cefoxitin or cefotetan with doxycycline. PCN allergic pt can receive clindamycin with gentamycin
    admission criteria include failure or intolerance of oral theray or fever or severe pain which may indicate the presence of an abscess.
    __________________________________________________ _
    protease inhibitors
    Nefinavir, Ritonavir, Lopinavir, indianavir, amprenavir, saquanivir, atazanavir, fosamprenavir, daruanavir, tipranivir

    protease inhibitors are highly effective anti-retorviral therapy
    they work by inhibiting the protease enzyme which is essential for the virus to be packaged as it buds off of a CD4 cell and leaves to infect other T cells

    All protease inhibitors cause hyperlipidemia and hyperglycemia in addition to being hepatotoxic, indianavir has a unique adverse effect of causing kidney stones nephrolithiasis

    protease inhibitors are a part of the best initial therapy for HIV when the CD4 count goes below 350 cells of the patient has has an opportunistic infection. the other most common question for protease inhibitors describes a pt who develops either an increased lipid level or kidney stones and asks which medication should be stopped
    __________________________________________________-
    Dapsone
    this is a sulfonamide antibiotic which is very selective against leprosy and is an alternative med for pneumocytis pneumonia
    it is a folic acid inhibitor
    the most common dverse side effect is a rash, it can also cause hemolysis with glucose 6 phosphate dehydrogenase deficiency. Methemogloblinemia can also occur, aplastic anemia can happen with all folic acid inhibitors.
    Dapsone is used to rx the following conditions
    PCP and toxoplasmosis prophylaxis where the pt is intolerant of TMP/SMX
    Leprosy
    Autoimmune skin disorders like Bullous pemphigoid and lichen planus and dermatitis herpetiforms.. which is a bit of shock to me as I often thought those were treated with hig dose steriod.. eh Dapsone it is!
    __________________________________________________ ___________

    all right not for diagnostics still cardio
    Persantine Thallium

    is a form of exercise intolerance stress testing. It is non-invasive method of determining mycoardial perfusion that uses med in lieu of physical exercise
    It is also called dipyridamole is a phosphodiesterase inhibitor that dilates the coronary arteries and increases O2 flow to the myocardium. Thalium is a radioisotope that shold be picked up by normal myocytes. Persantine increases thalium uptake in the normal areas of the heart and less in the disesed parts of the heart.
    Answer persantine thalium when pts cannpt exercise sufficiently to do a standard exercise tolerance test. the pt has a history of chest pain but you are not certain if it is ischemic in nature. It can also be used for preoperative screening.
    Persantine can't be used for pts with asthma, COPD or emphysema, because it provokes bronchospasm. stress testing in general should not be done with an acute infarction or unstable angina.

    use a dobutamine echo stress test when you can't use persantine
    the single most accurate tst for myocardial perfusion is the coronary angiogram.
    __________________________________________________ _
    Sestamibi testing

    this is a nuclea stress test also named Tc99m
    TC 99m is a labeled compounds require intact perfusion and viable myocardial cells. essentially a myocardial perfusion agent is indicated for detecting coronary artery diseease. Diseased or infarcted myocardium picks up less of the nuclear isotope. Ischemic myocardium re-perfuses at rest. The defect is reversible infarcted myocardium does not change with rest.
    the bottom line is sestamibi is a nuclea stress test that is used on obese pts and female pts with very large breasts.
    __________________________________________________ ____

    stress exercise tolerance
    this is an attempt to detect myocardial ischemia. pts without the need for cornoary angiography. it is used when the diagnosis of ischemia is not clear in a person with chest pain

    the pt exercises to >80% max heart rate bases on a maximum rate of 220-age. The EKG is observed for signs of ischemia. such as St segment depression. Hypotension, light headedness, and chest pain are taken as positive tests as well.

    Stress test is answer
    when pain is atypical and the story is equivocal
    post MI to determine the need for angiography
    with pt on meds to determine the amounts of meds are sufficient to prevent ischemia.
    Abrnomal stress test is confined with angiography
    __________________________________________________ _______
    stress echo

    is used to confirm the suspicion of coronary artery dz and estimate its severity
    ut uses both 2D echo and doppler echo to detect ishcmia. MI gives decrease systolic contration of the ischemic area, called regional wall motion abnormality. the pt is asked to exercise using a tradmil or bicyle and if can't exercise dobutamine can be ised to increase myocardial demand.
    a positive test consits of new regional wall motion abrnomalities a decline in ejection fraction and an increase in end systolic volume with sress on stress echo.

    answer stress when case is equivocal for ishcemic heart dz and the EKG is so abnormal that you can't read it for ischemia.
    __________________________________________________
    Thalilium stress testing
    this is the most accurate method of assessing myocardial perfusion without using angiograpy.
    Thallium-labeles red cells are injected into the pt's blood stream. a Thallium scan provides a view of the blood flow into the heart muscle. stress thallium is an alternative to stress echo. the indication for stress thallium are
    When resting EKG changes make exercise EKG difficult to interpret. such as the presence of LBBB, baseline St changes, left ventricular hypertrophy, pacemaker, or the effect of digoxin on the EKG,
    to localize the region of ischemia
    to assess revascularization following bypass of angioplasty

    the most accurate test for myocardial perfusion is a coronary angiogram!
    The Medical student Review

    Text without context is pretext
    If your opponent is of choleric temperament, seek to irritate him 44845203 1 - The Medical student Review


  9. #7
    جوري's Avatar Full Member
    brightness_1
    Soldier Through It!
    star_rate star_rate star_rate star_rate star_rate star_rate star_rate star_rate star_rate star_rate star_rate
    Join Date
    Jul 2006
    Location
    من ارض الكنانة
    Gender
    Female
    Religion
    Islam
    Posts
    27,759
    Threads
    1260
    Rep Power
    259
    Rep Ratio
    89
    Likes Ratio
    23

    Re: The Medical student Review

    today's 5/5 we are finishing cardio and moving on with endocrine with the last of the first 5, still on ID in pharmacology

    Diagnostics/ cardio

    Title table testing
    is used to evaluate unexplained syncope. It is the measurement of blood pressure and pulse at various angles while the patient is tilted up and down while lying on a table

    blood pressure and pulse are measured before and after moving the patient into different positions. The development of syncope, dizziness, abnromal drops in blood pressure constitute an abnormal test . Isoproterenol and nitroglycerin can be used as provocative testing.

    table tilt is done when there is unexplained syncope, particularly when there are signs of orthostasis such as an inappropriate bradycardia when standing up.
    _____________________________________________
    Transesophageal echocardiography TEE

    this is an extremely sensitive method os assessing cardiac structures. TEE is particularly sensitive at assessing posterior structures of the heart such as diseases of the aorta, example, dissection or aneurysm. Atrial thrombi, patent foramen ovale, or vegetations on the valves for the diagnosis of endocarditis . TEE is used when limited Transthoracic echocardiogram is inadequate.

    TEE is performed by placing a sonographic transducer through the mouth of the esophagus
    TEE is used to assess valvular heart disease as well as before the cardioversion of atrial fibrillation to exclude thrombi in the left atrium

    The most accurate diagnostic test is a cardiac cath of the left heart.
    _______________________________________
    Transthoracic or 2D Echocardiogram TT
    E

    TEE is used to detect chamber size and function, valve abnormalities, intracardiac masses, pericardial effusions and aortic disease

    Images of the heart are obtained from a sonographic transducer placed over the anterior chest wall
    TTE is done to assess every patient with an ausculatory abnormality for the severity of valve dysfunction. TTE has become the second most frequently done test to evaluate the heart after EKG.
    TEE is done to assess every pt with CHF to obtain ejection fraction. Therapy for CHF differs markedly based on whether there is systolic or diastolic dysfunction.

    Both transesophageal echo and coronary angiograms are more sensitive than the TTE
    __________________________________________________ _

    Troponin levels
    troponin is an enzyme that is released only from injured myocardium. Troponin is extremely specific for myocardium and is rarely derived from other sites in the body.

    Troponin levels are the answer in most acute coronary syndromes as the most accurate means of assessing myocardial ischmia or infarction. Answer troponins in all patients in the emergency dept with acute severe pain. this is true even if the EKG is normal.
    Myocardial ischemia as found by elevated levels of troponins is confirmed with EKG, echocardiography and angiography.
    ___________________________________________

    last in diagnostics today is first in Endocrinology
    we start off with
    antimicrosomal Antibodies
    Antithyroid peroxidase Anti-TPO
    Antibodies
    Antithyroglobulin antibodies

    These antibodies are associated with Hashimoto's thyroiditis and graves' disease. If hypothyroid, if they are positive, you will move straight to treatment with synthroid because etiology of the hypothyroidism has been confirmed.
    They are the answer once you have proven hypo- or hyperthyroidism. They are not to diagnose the thyroid function state. they are to confirm the autoimmune etiology of thyroid state,.
    You expect the levels to be high with autoimmune thyroiditis. this is whether the person is hypo or hyperfunctioning. nutritional problems and synthroid abuse will give low levels of the antibodies.
    _______________________________________________
    pharm still on infectious

    43 year old female comes to the ER having fever, headache, photophobia for the last six hours. Her lumbar puncture shows
    1,400 white cells that are predominately neutrophils

    Best initial therapy for bacterial meningitis is ceftriaxone, vancomycin and steroids.
    ceftriaxone and vanc both inhibit cell wall of bacteria and are bactericidal. Ceftriaxone is a beta lactam antibiotic that inhibits peptidoglycan cross-linkng. vancomycin is glycopeptide antibiotic.
    Glycopeptides inhibit the disaccharide precursors to peptidoglycans. Steriods presumably decrease inflammation of the central nervous system and decrease mortality in acute bacterial meningitis.

    Ampicilin is added to the other meds when the pt may have Listeria infection. Listeria monocytogenes occurs more frequently in the elderly, neonates, and those who are immunocompromised, such as pts on steriods, chemotherapy, asplenic pts and those with HIV.
    __________________________________________________ __________

    Man has been in the hospital for the last ten days on a ventillator f pneumonia. he develops diarrhea, and the stool toxin is positive for c. diff

    2- man is readmitted for recurrence of colitis from C.diff

    Metronidazole is the best initial therapy for c.diff, should be used orally, IV metro should only be used if oral rx isnt possible
    Metronidazole gives a disulfiram like rxn with alcohol, resulting in vomiting. It can also cause metallic taste in the mouth and peripheral neuropathy.

    The small percentage of pts who do not respond to metro should be switched to oral vancomycin. vanco can't be used IV for colitis.

    recurrent episodes can be treated with oral metro. is the pt responded to it the first time.
    _________________________________

    48 year old injection user comes to the ER with fever, rash, on P.E he has a pan-systolic murmur heard best over the left lower border. The murmur increases on inspiration

    tricuspid valve endocarditis occurs more often in the injection drug users because of the constant bathing of the right side of the heart with staphylococus. Tricuspid regurg is best heart at the lower left sternal border and all the right sided lesions will tend to increase in loudness with inspiration, which increases venous return to the right side of the heart.
    The best intial therapy with presumed endocarditis is to draw blood cultures and start empiric therapy for endocarditis is vancomycin and gentamycin
    vanc. inhibits the cell wall and aminoglycosides and gentamycin inhibits the ribosome.
    vanc can result in 'red man syndrome' froom a rapid infusion. this is flushing from histamine release by the vanco. aminoglycosides are nephrotoxic.
    _________________________________________________

    vomiting and severe diarrhea ensue after a sushi meal, in addition to rash and angioedema, wheezing and flushing

    this is scombroid fish poisoning charcterized by suden onset of diarrhea, wheezing and flushing, scombroid fish poisoning occurs from certain types of fish such as tuna, mackerel, bonita, and mahi-mahi. bacteria within the fish result in the production of histamine. scombroid fish poisoning has the fastest onset of all forms of food poisoning with symptoms occuring within mins of ingestion.

    RX for scombroid fish poisoning is with antihistamines such as diphynhydramine. Epinephrine is used in sevre cases.
    __________________________________________________ ______
    a woman with oncyomycosis of her toenail, the KOH of a scraping of the nail shows fungal hyphae
    Terbinafine is the best theray for onychomycosis. it has greater efficacy than griseoflulvan. Terbinafine is used for 6 weeks for fingernails and 12 weeks on toenails. itraconazole is also superior to griseofulvan but not as efficatious as terbinafine

    terbinafine works by inhibiting squalene epoxidase, this interfers with the production of ergesterol, whch is necessary to make the fungal cell wall. Squalebe accumulates within the cell wall which may account for terbinifines fungacifal activity.

    The most common adverse effects of terbinafine are hepatotoxicity and taste disturbance..
    The Medical student Review

    Text without context is pretext
    If your opponent is of choleric temperament, seek to irritate him 44845203 1 - The Medical student Review


  10. #8
    جوري's Avatar Full Member
    brightness_1
    Soldier Through It!
    star_rate star_rate star_rate star_rate star_rate star_rate star_rate star_rate star_rate star_rate star_rate
    Join Date
    Jul 2006
    Location
    من ارض الكنانة
    Gender
    Female
    Religion
    Islam
    Posts
    27,759
    Threads
    1260
    Rep Power
    259
    Rep Ratio
    89
    Likes Ratio
    23

    Re: The Medical student Review

    today we continue on with endocrinology in diagnostics and still on infectious disease in pharm

    24 hour urine cortisol level
    when collected for 24 hrs, urinary cortisol excretion is the most sensitive indicator of endogenous hypercortisolism and is the most sensitive test for Cushing's syndrome.
    Cortisol is a glucocorticoid hormone produced in the adrenal cortex
    look for a question with a case with truncal obesity buffalo hump striae, and easy bruising, answer 24 hour urine cortisol or overnight dexamethasone suppression test as best initial
    _____________________________________________
    ACTH (cosyntropin) stimulation test

    this test evaluates adrenal function, and is the best initial test for adrenal insufficiency. baseline cortisol levels are taken, then ACTH is administered and follow up cortisol levels monitored. If cortisol levels rise, he adrenal gland is functioning. If cortisol is low think primary adrenal insufficiency.

    look for a fatigued patient with skin hyperpigmentation, hyperkalemia, slight metabolic acidosis, and taste for pickle juice (high salt content) also look for ICU patient with unexplained hypotension.
    _______________________________________
    C-Peptide
    serum measurement of C-peptide is the best test to distinguish endogenous from exogenous hyperinsulin in pts with hypoglycemia.
    low values of C petide in pts with hypoglycemia represent factitious insulin injection. The pt is abusing insulin.
    High C peptide values represent endogenous insulin production, such as that indiced by sulfonureas or from an insulinnoma.

    look for pts with unexplained hypoglycemia, high insulin levels, and multiple hospital visits. Answer C-peptide as the next best step to determine etiology.
    _______________________________________________
    Fine needle aspiration of the thyroid
    a sample of the thyroid cells collected by a needle inserted into the thyroid is the initial test for all solitary nodules unless the patient is thyrotixic
    look for a case of palpable nodule and normal thyroid function tests
    this however isn't what to do for a patient with a solitary nodule who are thyrotoxic. in such pts a SCINTISCAN (I121 scan) should be done first to see if the nodule is hyperfunctioning (hot) or hypofunctioning (cold). hyperfunctioning nodules are almost never malignant.
    ________________________________________________
    oral glucose tolerance
    this is a measurement of serum glucose levels at baseline and two hours after ingestion of an oral load of 75 grams of glucose
    a positive test means that the glucose level 2 hours after the ingestion of the oral glucose load is >200mg %. this is one of the methods of diagnosing diabetes. The ordinary method is to find two fasting glucose levels to be >126mg/dl
    the strongest indication for an oral glucose tolerance test is in screening for diabetes during pregnancy. because of the short duration of pregnancy, the potential for marked worsening over a short period of time and harmful side effects of diabetes on the fetus. this is the single most urgent time tim make a prompt diagnosis of diabetes. Oral glucose tolerance testing is like putting your pancreas on a treadmill to evoke subtle amounts of disease not detected by ordinary test.
    __________________________________________________ _____
    now pharm
    60 year old man comes to the ER with S.O.B, cough, purulent sputum, temp 103 F. his WBC is 18,000. His pO2 is 65 on room air, CXRAY shows infiltrate in the right upper lobes

    the most important factor in determining the need for admission in pneumonia is the severity of hypoxia. The white cell elevation, the purulent sputum don't denote then pt needs admittance. CXRAY abnormality by itself doesn't mean the pt needs admission. It is not the number of lobes involved rather the level of hypoxia, hypotension, tachycardia, or confusion tat idnciates a need for IV therapy.
    outpatient pneumonia is best treated with oral macrolide, such as azithromycin, or clarithromycin. if there is serious underlying lung disease, respiratory fluroquinolones such as levofloxacin, gatifloxacin or moxifloxacin are the best first choice.
    in patient pneumonia is treated with the combination of ceftriaxone and azithromycin or with a fluroquinolone.
    __________________________________________________ ____

    a man comes to the clinic with genital ulcer and surrounding lymphadenopathy. the ulcer i painless and heaped up. indurated edges, his darkfield examination and rapid plasma reagin is negative

    a single intramuscular injection of benzathine PCN is the best initial therapy for primary or secondary syphillis. sx include a chancre, rash, lymphadenopathy, or condylomata laa. RPR can be negative in primary syphillis
    PCN allergic pts with primary and secondary syphillis are best treated with doxycycline or a macrolide, doxycycline is superior in efficacy.
    besides allergic rxn, the most common adverse effect of RX is Jarisch-Herxheimer reaction. this consists of fever, rash, mylagias, and a headache . this is thought to result from the release of lipopolysaccharides from dying spirochetes. and is treated with ASA.
    __________________________________________
    24 year old pregnant woman is in the clinic fo routine pre-natal care. her VDRL or RPR is reactive at a titer of 1:164, and the FTA-ABS is reactive, she has no sx

    secondary syphilis of indeterminate duration is referred to as 'latent' syphillis, when the serology for syphilis is positive but the duration of the infection is unknown, the syphilis is latent and treated with three intramuscular injections of benzathine pcn give at weekly intervals. the same rx in pregnancy

    pregnant pt who is allergic to PCN should undergo desensitization. true for any stage of syphillis
    _________________________________________________
    24 year old female with vaginal itching, and dyspareunia. her wet mount shows clue cells and ph 6.5

    34 year old woman has discharge and wet mount shows motile forms

    the sx of bacterial vaginits are clue cells and high ph. Normal vaginal PH is <4.5. Metronidazole or clindamycin are the best initial therapy for BV and can be used orally or locally

    Bacterial vaginits in pregnancy is best treated with oral mes to prevent pre-term delivery. Oral metronidazole is not dangerous in pregnancy. untreated vaginitis is.

    case two trichomoniasis is treated with only oral meds, oral metronidazole or oral tinidazole are best.
    pregnant women with trichmoniasis should receive oral metronidazole.. there is no proven tetragenocity

    will finish the lat case after I break fast insha'Allah
    __________________________________________________ _____
    ok post breakfast last caveat for the day
    25 year old woman who is HIV positive finds out she is pregnant, she is on zidovudine, lamivudine and neflanavir

    a woman who tests positive for HIV as a part of routine prenatal care, her viral load is 300,000, and her CD4 is 85

    a patient who is already on antiretroviral medication for her own health with a low CD4 count in the past should be continued on the same therapy, don't stop any antivirals evern during the first trimester if the mother's life is dependent on the medication.
    for case two a woman with low CD4 count (<350) or high viral load (>50,000) should be started immediately on antiretroviral therapy. even if she is pregnant. Antiretrovirals should be started even in the first trimester of pregnany if the woman needs medication for her own health. Triple therapy with two nucleoside reverse transcriptase inhibitors and a protease inhibitor should be used and continued life long, even aftr delivery is completed.

    A ceasarian section should be performed if the patient's viral load is not fully controlled to a lavel <1,000 copies. the majority of perinatal transmission occurs at the time of delivery
    Efavirenz is a non-nucleoside reverse transcriptase inhibitor. it is specifically contraindicated in pregnancy because of the risk of neural tube defects,.

    the baby should receive zidovudine (AZT) syrup for at least six weeks after delivery.
    Last edited by جوري; 09-07-2008 at 12:03 AM.
    The Medical student Review

    Text without context is pretext
    If your opponent is of choleric temperament, seek to irritate him 44845203 1 - The Medical student Review


  11. #9
    جوري's Avatar Full Member
    brightness_1
    Soldier Through It!
    star_rate star_rate star_rate star_rate star_rate star_rate star_rate star_rate star_rate star_rate star_rate
    Join Date
    Jul 2006
    Location
    من ارض الكنانة
    Gender
    Female
    Religion
    Islam
    Posts
    27,759
    Threads
    1260
    Rep Power
    259
    Rep Ratio
    89
    Likes Ratio
    23

    Re: The Medical student Review

    Today it will be 10 and 10 insha'Allah to make up for tardiness of yesterday ( we had family for iftar and I did nothing)

    Still on endocrine for diagnostics

    Beta-Human Chorionic Gonadotropin (B-HCG)

    this is pregnancy test
    urine and/or serum measurements of B-hCG is the best initial test for any woman of reporiductive age complaining of a missed period (amenorrhea), delayed period, vaginal bleeding or abdominal pain, any unexplained abdominal pain in a woman is an ectopic pregnancy until you have the B-hCG
    _________________________________________________

    5-HIAA

    24 hour urine level of 5-HIAA is the best initial diagnostic test for carcinoid syndrome. 5-HIAA is the end product of sertonin metabolism

    pt might present with episodic diarrhea, and or hypotension associated with unexplained flushing or wheezing.
    the most accurate test for carcinoi syndrome is a biopsy. Although a CT and MRI scaning can localize the disease, they are not as accurate as the tissue diagnosis you obtain from a biopsy
    __________________________________________________ ____

    17-Hydroxyprogesterone level
    this is the best initial test to diagnose 21-hydroxylase deficiency or congenital adrenal hyperplasia.
    17-hydroxyprogesterone in a patient presenting with one of the following syndromes

    female infant with ambiguous genitalia (simple virillizing)
    infant with hyponatremia, hyperkalemia and hypotension (salt-wasting syndrome)
    young woman with hirsuitism, acne, and irregular menses
    child wih precocious puberty.
    __________________________________________________
    IGF-1 (Insulin- like growth factor-1)

    this is the best initial test for acromegaly
    IGF-1 is a somatomedin secreted from the liver. Growth hormone when combined with thyroid hormone stimulates linear skeletal growth in children through IGF-1. it stimulates protein synthesis in muscle (anabolic effect) in children and the release of fatty acids from adipose tissue (catabolic effect) in adults. Growth hormone secretion is pulsatile, has a short half life and reaches maximum life in the middle of the night. a single test for GH level is not accurate. IGF-1 levels are more stable an reliable.
    The best confirmatory test for acromegaly is the oral glucose tolerance test. Glucose normally suppresses GH. in acromegaly glucose doesn't suppress GH
    __________________________________________________ ___

    Pro-Insulin assay
    is used to distinguish autoimmne causes of hypoglycemia from insulinoma
    high levels of pro-insulin represents insulinomas
    answer a pro-insulin level when you see a case of recurrent fasting hypoglycemia, symptoms of weakness and a high c-peptide levels
    __________________________________________________ ______
    progesterone challenge
    oral progesterone of 10mg dmily for 5 days is used in amenorrheic women after pregnancy is excluded to determine is the cause of the amenorrhea is due to outflow tract obstruction or secondary to the hypothalmus and pituitary. Look for a woman who has no periods and a negative hCG.
    if bleeding occurs following the challenge, it indicated that there is adequate estrogen and that the outflow tract exists.
    the next best test to do is to check the FSH, LH, prolactin and TSH to rule out a hypothalmaic or pirutitary etiologu
    if no bleeding occurs, the next best test is to evaluate for an outflow obstruction with ultrasound, and with MRI if ultrasound isn't definitive.
    __________________________________________________ _______

    Metaiodobenzylguanidine (MIBG) scan
    this is used to detect for pheochromocytoma if the blood urine testing (VMA, catecholamines, metanephrine) are positive and the scans are nondiagnostic.
    MIBG is injected into the body. MIBG is an agent similar to norepinephrine and should be taken up by adrenergic tissue. the MIBG scan allows visualization of potential adrenal masses.
    Answer MIBG scan when you see a patient with palpitations, intermittent tachycardia, and biochemical evidence of pehocrhromocytoma (e.g., catecholamines, metanephrines) when the CT and MRI can't visualize the lesion. you can't remove a pheochromocytoma and cure it, if you can't find it!
    __________________________________________________ _______

    RAIU (radioiodine uptake)
    the measurement of orally ingested radioactive iodine (I121) that accumulates in the thyroid gland is used to differentiate Graves toxic multinodular goiter from thyroiditis/factitious.
    RAIU is increased in Graves disease and wth hot nodules (toxic multinodular goiter, toxic solitary nodule)
    RAIU is decreased when thyroiditis induces a thyrotoxic state or when a person is abusing thyroid hormone (thyroiditis factitia)
    Answer RAIU scan when there is a case of hyperthyroidism. The Question will include a high T4 level, for instance and will ask what is the next step to determine the etiology.
    __________________________________________________ __

    TSH
    Thyroxine (t4)
    Thyroglobulin

    serum levels of thyroid stimulating hormone (TSH) and throxine (t4) are measured to assess thyroid function (both hypofunction and hyperfunction). Thyroglobulin is used to monitor recurrence in pts who have been treated for papillary or follicular cancer of the thyroid.
    TSH is produced by the pituitary to stimulate the thyroid. T4 is once of the principal metabolism-inducing hormones produced by the thyroid gland, and thyroglobulin is an iodine-containing glycoprotein synthesized by the thyroid gland from which thyroxine T4 is derived. Thyroglobulin is the storage form of thyroid hormone. Thyroid-binding globulin is the transport protein.
    TSH and T4 are best initial tests
    thyroglobulin is to follow the response to the treatment of thyroid cancer.
    __________________________________________________ ______
    Venous blood gas

    use VBG for follow up of diabetic ketoacidosis (DKA), a VBG could replace the arterial samle for assessing the degree of acidosis and hypercarbia when all you need is ph and not the pO2 while avoiding an additional painful procedure
    ________________________________________________

    PHARM
    for pharm I'll be finishing infectious disease and moving on to neurology
    ________

    an HIV + male comes with several weeks of worsening difficulty and pain (odynophagia) with swallowing. His CD4 is 85

    start treatment with fluconazole. Dysphagia and odynophagia in a person with AIDS and CD4 count <100 is often from esophageal candidiasis that there is no need for endoscopy, therefore treatment should be started right away.

    imidazoles and triazoles such as fluconazole, impair the biosynthesis of ergesterol for the cytoplasmi membrane.
    Azole antifungals are extremely benign. After prolonges use at high dose they can be hepatotoxic.
    if the condition doesn't improve with fluconazole, endoscopy should be performed to diagnose rarer causes of esophageal lesions in HIV such as Herpes or CMV.
    __________________________________________________ ______

    34 year old man has been newly diagnosed with AIDS, His CD4 is 20 and his viral load is 450,000. He is currently asymptomatic

    antiretroviral therapy should be started in most pts with CD4 cells <350 and in all pts with CD4 cells <200 two nucleoside reverse transcriptase inhibitors (NRTIs) with either efavirenz or a protease inhibitor (PI) should be started. Pneumocystis carinii penumonia (PCP) prophylaxis with TMP/SMX should be given when the CD4 drops below 200. Azithromycin for mycobacterium avium intracelulare (MAI) prophylaxis should be given to pts with CD4 cells <50

    NRTIs inibit the transcription of HIV RNA into DNA, they terminate the synthesis of DNA chains. Normally, the single-stranded RNA virus is transformed into DNA, which is integrated into the host cell genome NRTIs do not affect host cell DNA plymerase. PI's inhibit the viral protease enzyme that is crucial to maturation of HIV, allowing budding from host cells.
    All NRTIs can cause lactic acidosis. All PI's can cause hyperlipidemia and hyperglycemia. ZIdovudine can cause aneia. Didanosine and stavudine can cause pancreatitis and neuropathy. Tenofovir can cayse renal insufficiency and fanconi's syndrome.
    _______________________________________
    a man with peptic ulcer disease is admitted for a perforation, he is febrile, hypotensive and tachycardic. He is started on imipenem and gentamycin, three days later his creatinine rises to 3 and he has a seizure

    Gentamycin, tobramycin and amikacin are aminoglycoside antibiotics. they are primarily effective against gram negative bacilli. Imipenem, meropenem and ertapenem are carbapenem antibiotics. they cover the aerobic gram negative bacilli covered by aminoglycosides as well as anerobes. The anerobic coverage of carbapenems is equal to that os metronidazole. in addition, carbapenems cover all methicillin or oxacillin sensitive staphylococci and streptococci.

    carbapenems inhibit cell wall synthesis in the same way PCNS by inhibiting peptidoglycan synthesis. They binf to PCN-binding proteins. Aminoglycosides inhibit protein synthesis by binding to the 30s ribosomal subunit.
    Aminglycosides lead to nephrotoxicity, and ototoxicity. rarely they cayse neuromuscular blockade and produce mysthenia like weakness. Carbapenems can cause neurotoxicity.
    the pts seized as an adverse effect of the imipenem. the level of imipenem rose due to his renal impaired function and a toxic level caused a seizure.
    __________________________________________________ __
    23 year old woman with dysuria and WBC in her urine started on TMP/SMX, she develops a rash and is switched to cipro

    43 year old woman with pyelonephritis and E.coli bactermia develops anaphylaxis to ampicillin, she is switched to aztreonam

    Cirpo is broadly active against the majority of gram negative bacilli. in addition all fluoroquinolones are active against mycoplasma, legionella and chlamydia pneumonia. Aztreonam, a mono bactam the only drug in its class active against gram negative bacilli. There is no cross reactivity between aztreonam and PCN. Hence, aztronam can be used even if there has been anaphylaxis with PCN.

    Quinolones, such as cipro, exhibit concentration dependent bactericida activity by inhibiting the activity of DNA gyrase and topoisomerase, enzymes essential for bacterial DNA gyrase and topoisomerase enzymes essential for bacterial DNA replication, aztreonam is a beta lactam antibiotic and inhibits cell wall synthesis.

    quinolones can rarely be associated with central nervous sysem CNS toxicity. inhibition of chondroblasts can lead to achilles tendon rupture. this is why they are contraindicated in pregnancy and in children. Gatafloxicin can cause hyperlycemia, occasionally prolongs the QT interval, Aztreonam is generally safe wit few side effects.
    __________________________________________________ __________________________________

    64 year old man with diabetes develops an ulcer over his tibia. The X-Ray is normal, but the MRI shows osteomyelitis. a bone biopsy is performed, which grows an organism

    osteomyelitis can't be effectively treated without a biopsy of the bone. Staphylococci accounts for only 50-70% of isolates. There is no way to determine the sensitivity of the organism without a biopsy. treatment is as follows
    Staphyloccocus aureus: oxacillin sensitive organismss are best treated with nafcillin or oxacillin. ceftraiaxone or cefazolin can be used as well. if the pt suffers a rash due to PCN, cephalosporins can be safely used. Oxacillin organisms are treated with vancomycin, liezoli, or daptomycin.
    Gram negative bacilli. is the organism is sensitive the drug of choice is a fluroquinolone, such as sipro, quinolones can only be used orally
    The ESR can be used to follow the response to therapy. Most pts are afebrile at the beginning oftherapy and have a normal WBC count, so those measures can't be used to monitor therapy.
    __________________________________________________ ____________________________

    in a congressional room 10 envelopes with white powder arrives marked 'pls inhale upon opening'
    inhalational anthrax spores, best prophylactic meds are cipro, or doxycycline, Amoxixillin is a second line agent when quinolones of doxycycline are contraindicated.
    __________________________________________________ ________________________________

    53 year old alcholic with cirrhosis an ascites is admitted for fever and abdominal pain, a paracentesis reveals 1.200 WBC that are 80% neutrophils

    spontaneous bacterial peritonitis (SBP) is treated when the ascitic fluid reveals either total blood cells >500 or neutrophils >25-
    cefotaxime is the treatment of choice for SBP
    prophylaxis is indicated for the following conditions
    pt has experienced previous SBP
    Ascites occurs with variceal hemorrhage
    Ascitic protein content <1.0 grams/dl
    the best prophylactic medication is either norfloxacin or TMP/SMX
    __________________________________________________ ________________________________
    child comes to the ER with sudden onset of severe sore throat and stridor. voice is muffles, she is drooling, P.E in the operating room with direct fibroptic laryngoscopy reveals epiglottitis

    in cases of epiglottitis, securing the airway with naotracheal intubation is the most urgent step. The airway may close off abruptly and the examination should be done in the OR

    antibiotic treatment for epiglottitis is with ceftriaxone and clindamycin.

    woman comes in for genital warts on her labia majora. They are not uncomfortable but she would like them removed

    condylomata accuminata, can be treated with a large number of methods, equal in efficacy. these are cryotherapy, podophyllin, laser removal, trichloacetic acid and imiquimod. Imiquimod is a local immunostimulant that acts by local cytokine induction and leads to sloughing off of the lesion very slowely over a period of months.
    all therapies with exception of imiquimod can damage surrounding healthy tissues.. podophyllin is teratogenic.
    __________________________________________________ ___________________________________

    ok we are officially done with infectious and now have moved on to Neurology we start by Dopamine agonists

    carbergoline
    Bromocriptine
    Ropinerole
    Pramiperxole
    Pergolide

    Dopamine agonists are the correct medications for the following conditions

    Parkinson's disease, these medications are less potent but have fewer side effects when compared with levodopa/carbidopa.
    Restless leg syndrome
    prolactinoma, best initial over surgery

    adverse effects
    Nausea, vomiting, sleepiness, orthostatic hypotension, confusion and hallucination. Pergolide was taken off the market for adverse cardiac effects (valve regurgitation)
    __________________________________________________ _____

    Amantadine

    best initial therapy for
    Acute influenza A, within 48 hrs of onset of sx, Amantadine is inferior to ossltamivir
    mild parkinsons disease (PD) is characterized by predominately a tremor, and the pt retains the ability to perform activities of daily living

    Amantadine and rimantadine both inhibit the replication of the influenza A virus, Amantadine enhances dopamine release from the substantia nigra and prevents re-uptake.

    adverse
    it may cause confusion, hallucination, nightmares, depression and restlessness, also livedo reticularis can occur.


    ___
    Last edited by جوري; 09-09-2008 at 01:39 AM.
    The Medical student Review

    Text without context is pretext
    If your opponent is of choleric temperament, seek to irritate him 44845203 1 - The Medical student Review


  12. Report bad ads?
  13. #10
    جوري's Avatar Full Member
    brightness_1
    Soldier Through It!
    star_rate star_rate star_rate star_rate star_rate star_rate star_rate star_rate star_rate star_rate star_rate
    Join Date
    Jul 2006
    Location
    من ارض الكنانة
    Gender
    Female
    Religion
    Islam
    Posts
    27,759
    Threads
    1260
    Rep Power
    259
    Rep Ratio
    89
    Likes Ratio
    23

    Re: The Medical student Review

    I was struck and aggrieved by Snakelegs passing yesterday--

    I'll be doing 10 of each today to make up for yesterday.

    we are on pharm (Neurology) and Endocrine diagnostics
    let's start with Pharm
    ___
    45 year old woman evaluated because of painful contracted muscle. There is marked hypertonicity and hyperreflexia.

    Baclofen and Tizanadine can be used for spasticity. Diazepam and dantrolene can also be used but are less effective
    Vaclofen is an analog of GABA. the precise mechanism of relieving spasticity isn't known. Tizanadine is centrally acting alpha adrenergic agonist but does not lower pressure. Tinazadine inhibits pre-synaptic motor neurons.
    The adverse effects of baclofen are dizziness, lighteheadedness, drowsiness, and weakness. Tinazidine causes dizziness, lighthededness and dry mouth
    the rate of progression of MS can be slowed with beta interferon and copolymer I (glatirimer)
    __________________
    A man is evaluated for a motor neuron dz in which he has fasciculations, muscle wasting, hyperreflexia, weakness, and upgoing toes, but cognition is intact

    ALS is a combination of upper and lower motor neuron dysfunction. There is weakness, wasting, hyperreflexia, and fasciculations, but intact cognition, bowel, bladder, and sexual function. Als progression can be slowed down by riluole.
    Riluzole inhibits glutamate release presynaptically b blocking sodium channels.
    Riuluzole can case nausea, vomiting, weakness, and spasticity.
    ________________
    Lamotrigine

    used to treat the following conditions
    Seizure disorder, generalized or partial
    Peripheral neuropathy
    mood disorders, such as bipolar disorder
    Lamotrigine blocks the repetitive firing neurons by inactivating voltage dependent sodium channels. It may selectively influence neurons that synthesize glutmate and aspartate.
    Lamotrgine can cayse very severe skin reactions, such as steven's Johnsons syndrome.
    ______

    A man comes to the ER because of sudden, severe, sharp, pain in his face while he was eating. It started near the side of the mouth and progressed to the ear. It resolved after a few seconds and then recurred. Currently his sensory exam is normal

    caebamazepine is the best initial therapy for trigeminal neuralgia. it should be effective in more than 90% of cases. for those not responding. Phenytoin, baclofen or gabapentun maybe effective, surgery is performed if all meds fail

    carbamazepine works in a similar fashion to phenytoin. It blocks the sodium channels at therapeutic concentrations. and inhibits high frequency repetitive firing in the neurons in culture. It also acts presynaptically to decrease synaptic transmission.

    carbamazepine is also effective as first line medication for seizures and for peripheral neuropathy.
    carbamazepine can cause diplopia, ataxia and hyponatremia, rarely it brings on aplastic anemia.
    ____
    Levetricetam

    Levetricetam is effective in the treatment of seizures.
    it modifies the synaptic release of glutmate and GABA throigh action on vesicular function
    levitricetam has virtually no adverse effects and does not require drug monitoring. Some pts feel weak, sleepy and dizzy.
    ____

    pt with generalized recurrent seizure. there have been several short seizures over a few months that resolved spontaneously. There are no focal deficits, and all lab testing is normal.

    these meds are acceptable as initial treatment for generalizes seizures.
    it isn't clear which is superior

    Valproate oxcarbzepine
    Carbamazepine Phenobaribital
    Phenytoin Topiramate
    Lamotrigine

    the adverse effects are


    Valproate : weight gain, tremor, hepatotoxicity
    oxcarbzepine: vertigo, Ataxia, hyponatremia
    Carbamazepine : Hyponatremia, lethargy, neutropenia
    Phenobaribital: sedation
    Phenytoin : Gingival hyperplasia
    Topiramate : Glaucoma
    Lamotrigine : Stevens-Johnsons Syndrome

    _________________________
    62 year old man with diabetes comes to the office, he has been experiencing numbness and tingling in his feet for the last few months

    either gabapentin or pregabalin is the best initial rx for peripheral neuropathy, especially from diabetes or HIV. Both of these meds can be used for seizure as well.
    Both meds are analogs of GABA
    the most common adverse effects are somnolence, dizziness, ataxia, headache and tremor.
    peripheral neuropathy. Tricyclic antidepressants are also effective against neuropathy but can cause dry mouth, urinary retention and constipation because of their anti-cholinergic effect
    ________________

    tricyclic antidepressants
    Amitriptyline
    Imipramine
    Nortriptyline
    Desipramine
    Protriptyline

    these are indicated for
    Major depressive disorder
    obsessive-compulsive disorder, phobia, anxiety
    Attention deficit disorder
    chronic pain syndrome, and peripheral neuropathies
    nocturnal enuresis
    migraine headache prophylaxis
    TCA block the reuptake of amine neurotransmitters norepinephrine, and sertonin, leaving more of the neurotransmitters in the synapses.
    TCA can cause many anticholinergic adverse effects, even at therapeutic doses such as dry mouth, urinary retention,, constipation, and orthostasis. At toxic level they can cause QT prolongation, arrhythmias and seizures.
    ______________________________________________

    Catechol-Omethyltransferase (COMT)inhibitors:
    Tolcapone
    Entacapone
    Al capone (kidddddddddddding) just checking to see if you are awake hehe

    Tolcapone and entacapone are COMT inhibitors that extend the duration of levodopa replacement treatment for parkinsons' dz.
    COMT inhibitors prevent the degradation of levodopa. Levodopa is normally inactivated by COMT. COMT inhibitors keep levodopa levels higher for longer. Entcapone andtolcapone can't be used alone, they must accompany Levodopa replacement.
    this is for use in pts whose sx are progressing in spite increasing levodopa/carbidopa use.
    Adverse: Orthostatic hypotension, dyskinesias, confusion and fatal hepatic failure.
    __________________________________________________ _
    72 year old female comes to office with tremor difficult walking, she is able to take care of herself at home but foesn't like her increasing slowness. She has mild cogwheel rigidity ...
    Mild parkinson's disease is best treated with monoamine oxidase inhibitors MAOis selegeline, rasagilene tremor only
    <70 year olds, the anticholinergics trihexphenidyl and benztropine
    >70 yrs of age Amantadine
    Moderate disease
    Dopamine agonist: prmipexole and ropinerole: these have less postent, less adverse effects than levodopa replacement.
    Severe parkinsons
    Kevodopa/carbidopa (sinemet); if levodopa replacement doesn't control SX, a catechol-o-methyltransferase (COMT) inhibitor is added. Severe is defines as the inability to take care of oneself or to be employed.

    levodopa: carbidopa is added to bloc peripheral metabolism so more levdopa enters the brain
    selegeline and rasagiline. These MAO inhibitors block dopamine metabolism.
    Amantadine, increases dopamine release from the substantia nigra
    the adverse effects of these treatments is as follows
    levodopa: dyskinesia, the on'off phenomenon
    selegiline/rasagiline confusion, also, these medications interact with tricyclic antidepressants and SSRI's
    anticholinergics dry mouth, constipation and urinary retention.
    ________________________________
    Diagnostics we'll finish endocrine and move on to GI

    24 hour urine for metanephrine, vanillylmandelic acid (VMA) and catecholamines

    24 hour urine collection for metanephrine, VMA, and the catecholamines is the best initial test for pheochromocytoma.
    if VMA is elevated, then do a CT or an MRI of the adrenal glands as the next best thing for pheo
    if ct/MRI find nothing, a metaiodobenzylguaniidine (MIBG) scintigrapy would be the next best option. This is a norepinephrine analog that concentrates in the adrenals an pheochromocytomas.
    Pheochromocytomas is associated with MEN II and MEN III.
    look for a case of episodic HTN, particularly with flushing and diarrhea.
    _____________
    diagnostics GI

    03 06 0817 01 - The Medical student Review

    this is an abdominal x-ray
    the abnormality is a small bowel obstruction, there are multiple air-fluid levels seen throughout the bowel
    the most accurate test for small bowel obstruction is a combo od abdominal x-ray and CT scan. there is no blood test to diagnose obstruction, an there are no findings on endoscopy
    pt usually has abdominal pain and distension, hypoactive or absent bowel sounds, possible electrolyte abnormalities such as potassium, ca2+, mg 2+ disorders, and failure to pass stoll or flatus. this is normal after abdominal surgery.
    _____________________________

    1198 1 - The Medical student Review
    colon cancer on barium enema

    artar428357janufig2 1 - The Medical student Review
    diverticulosis on barium enema

    Barium Enema creates contrast picture to outline the lining of the rectum and colon. BE can be used in detecting colorectal cancer of the rectum and colon. BE can be used in detecting colorectal cancers, polyps, diverticulosis, and bowel obstruction and to evaluate the extent of inflammatory bowel disease.
    BE is inferior to colonoscopy if you are suspecting a lesion in the colon. Colonoscopy is also superior to enema in that it can directly biopsy lesions or remove polyps.
    Pts who may have peritonitis or bowel perforation must NOT undergo this test because the contrast material can leak into the peritoneum through the perforation. BE is also contraindicated in diverticulitis, in which it in creases the risk of perforation.
    __________________________________
    artmgi6433navafig1ab 1 - The Medical student Review

    this is an upper endoscopy showing a Barrett's esophagus
    this occurs when the distal stratified squamous mucosa is replaced by metaplastic columnar epithelium containing goblet cells. The metaplasia results as a form of defense from the persistent acid refluc that cells get exposed to. Barrett's esophagus is not reversible but is preventable with PPIs
    complications are ulcers, strictures and adenocarcinoma.
    __________________________________

    PairOfCapsules - The Medical student Review

    this is a capsule with a camera and a small transmitter. once swallowed it begins transmitting images of the inside GI to a receiver worn by the patient. the doctor can review 6 hrs with of pictures for any abnormality.

    this is the procedure of choice for suspected small bowel bleeding that was not detected on upper endoscopy.
    ___________________________________
    Amylase/Lipase levels
    those are initial tests in the diagnosis of acute pancretitis. Lipase is more specific for the pancreas. Amylase can be elevated from damage to the salivary glands, esophagus or small bowel.
    the patient presenting with the acute onset of severe epigastric pain radiating to the back. such pts may have a history of alchol abuse or gallstones.
    most accurate test for acute pancreatitis is an abdominal CT scan is more accurate than Ranson's criteria. Pancreatic necrosis on a CT scan is extremely specific for severe pancreatitis and is the main indicator of the need for pancreatic biopsy.
    ____________________________________
    Antiendomysial
    antigliadin antibodies

    serum assay for endomysial and antigliadin antibodies are made against wheat or gluten (gliadin) antigens and the villous (endomysial) linning of the small intestine. They are used to confirm dx of celiac disease
    celiac dz, presents with oily, greasy diarrhea, foul smelling stool witout evidence of cheonic pancreatitis.
    they are the tests to answer to confirm gluten sensitive enteropathy after sudan black stain has confirmed a fat malabsorpption
    small bowel biopsy is the most accurate test for celiac dz. it must be done to excluse lymphoma as well to dx the disease.
    _______________________________________

    Anti-mitochondrial antibodies (AMA)
    serum antimitochondrial antibodies is the best test to make a specfic diagnosis of primary biliary cirrhosis (PBC)
    look for a middle aged female presenting with itching (pruritus) and elevated alkaline phosphatase with a normal bilirubin.
    the most accurate test however is PBC is a liver biopsy.
    __________________________________________________ ________
    Anti-smooth muscle antibodies
    serum assay detection of anti-smooth muscle antibodies (ASMA) is the best initial test for autoimmune hepatitis.. you know I personally always forget this one too so many anti this and anti that, so remember liver is smooth and you'll remember anti smooth lol
    this is the test for a young woman with lover disease who doesn't drink or have inflammatory bowel disease or infectious hepatitis.
    Auto-immune hepatitis is associated with antinuclear antibodies (ANA) and antibodies to liver/kidney microsome type 1 (anti-LKM1)
    again liver biopsy is the most diagnostic .
    ____________________________
    Barium esophagus
    this is the best initial test for dysphagia. this is not in scenarios where there are clear signs of obstruction, for which upper endoscopy would be best initial. For achalasia barium esophagram shows a bird beak

    Classic achalasia Ba swallo - The Medical student Review at the esophageal junction. in diffuse esophageal spasm, barium esophagram shows a corkscrew

    ncpgasthep0284 F1 - The Medical student Review at the time of spasm.

    Barium esophagram is the most accurate test for esophageal rings, webs, and diverticuli.
    in achalasia, esophageal spasm, and nutcracker esophagus, the most accurate test is manometry. with cancer the most accurate is endoscoy for a biopsy.
    Last edited by جوري; 09-11-2008 at 04:43 AM.
    The Medical student Review

    Text without context is pretext
    If your opponent is of choleric temperament, seek to irritate him 44845203 1 - The Medical student Review


  14. #11
    جوري's Avatar Full Member
    brightness_1
    Soldier Through It!
    star_rate star_rate star_rate star_rate star_rate star_rate star_rate star_rate star_rate star_rate star_rate
    Join Date
    Jul 2006
    Location
    من ارض الكنانة
    Gender
    Female
    Religion
    Islam
    Posts
    27,759
    Threads
    1260
    Rep Power
    259
    Rep Ratio
    89
    Likes Ratio
    23

    Re: The Medical student Review

    Salaamz..
    sorry about yesterday, as usual with days missed will make up for it by doubling the number today, we start with pharm, finish Neurology and go on to oncology
    ___________

    Ethosuxamide

    is an anti-seizure med that is most effective in absent seizure
    it reduced low threshhold calcium currents in the thalamic neurons. The thalamus plays an important role in the generation of 3-HZ spike-and-wave rhythms typical of absent seizures

    most common side effect is drowsiness ..
    this is best used for a child who has a 10 to 30 second period of loss of consciousness. formerly called petit mal. there can be eyelid fluttering, but the child does not fall or convulse. the child abruptly stops activity. then just as abruptly resumes it with no postictal sx or knowledge that a seizure has occured. Absence seizure are genetic and occur predominantely in children.
    _____________________________
    Capsaicin

    is a cream that is used as an analgesic
    derived from red peppers and inhibits the release of substance P from sensory fibers. Substance P causes pain if you recall
    capsaicin causes burning and erythema at the site of application.
    capsaicin is a second line medication not the most effective for the following conditions
    postherpetic neuralgia
    osteoarthritis
    rheumatoid arthritis
    diabetic neuropathy.
    _________________________________

    26 year old woman comes to the ER with unilateral throbbing headache, she is nauseated and notices some bright flashes of light. she had several of these before. she takes OCP and is mensturating

    migraine headaches are aborted by dihydroergotamine and ergotamine combined with caffeine or one of the triptans. such as suatriptan, zolmitriptan, naratriptan or elitriptan.
    triptans and ergotamine are sertonin agonists that constrict large vessles, they are therefore contraindicated in coronary artery disese
    migraine headaches are best prevented by chronic use of beta blockers, such as propanaolol, the other prophylactic meds for migraine are valproic acid, gabapentin, calcium bloackers and tricyclic anti-depressants.

    __________________________________________________ ____

    60 year old man comes in for several severe unilateral, sharp headaches occuring over several days, they are associated with tearing on the eye, redness of the eye, ptosis, constriction of the pupil and rhinorrhea

    cluster headaches are aborted by inahling 100% oxygen. migraine medications such as sumatriptan or ergotamine may be effective as well

    cluster headaches are best prophylaxed with the use of lithium or prednison for sveral weeks, beta blockers are not as good a choice. the therapeutic benefit of propanolol to prevent headaches takes several weeks to become evident, and the cluster will be over by the time the beta blocker becomes effective.

    _______________________________________
    a 79 year old man is evaluated for increased forgetfullness and short term memory loss, he has no focal neurological defecits, the head CT shows bilateral symmetrical atrophy. the B12 level, rapid plasma reagin RPR and thyroid function tests are normal
    the best initial therapy for alzheimer's dementa is wih donepezil, galantamine or rivastigmine.
    the medications are ani-cholinesterase inhibitors. they act i the central nervous system to increase the level of ach.
    cholinsterase inhibitors can cause diarrhea and mild nausea and vomitting, this is because ACH acts as a promotility agent got the GI tract.
    _________________________________________

    34 year old man is admitted because of severe lower extremeity weakness which has developed over the past several week. it started in his feet and has ascended his upper legs, he recently had an episode of gastrointeritis. the deep tendon reflecex of his lefs are absent, Electromyograhy and nerve condition studies are normal

    Guillain Barre syndrome is treated with plasmapharesis or infusions or IV immunoglobulins. There is no benefit to combined therapy.
    Death from Guillain Barre occurs from progression to involvement of the respiratory muscles. monitoring of the vital capacity is needed to determine if the pt needs intubation.
    _____________________________________________

    A man comes to the ER after a continuous seizure for the last 30mins. There has been no response to repeated injections of benzodiazepines (lorazapam, diazapam)
    phenytoin or fosphenytoin us used to treat status epilepticus tat has not responded to benzodiazepines. Fosphenytoin is equally efficacious compared to phynytoin but has fewer side effects, such as hypotension and heart block.
    if fosphenytoin does not work then treatment with phenobarbital for the rare patient that does not respond to benzodiazepine, fosphnytoin or phenobarbital, then general anesthesia with midazolam, propofol or pentobarbital should be used. Neuromuscular blocking agents such as vecuronium or succinylcholine, will stop the mucle contractions but will not stop the seizure in the central nervous system.
    __________________________________________________

    42 year old man comes to ER dept with sudden onset of very severe headache. There is neck stifness as well, his BP is 170/120. The CT scan shows a subarachnoid hemorrhage.

    SAH is initially managed by controlling the blood pressure to systolic <160 to prevent further bleeding.
    Nimodipine is a calcium channel blocker that is used to prevent subsequent vasospasm, Nimodipine is the standard of care after SAH to prevent stroke.

    A cereberal angiogram is performed to guide operative therapy. Surgical clipping is performed to prevent rebleeding. Embolization of the site of aneurysm is useful and can be done by a catheter, a platinum wire is placed into the site of bleeding. embolization can be as effective as surigcally placed clip and doesn't require a craniotomy.
    __________________________________________________ _

    a man is evaluated for dilopia, and weekness. His anti-cholinesterase antibodies are positive. single fiber electromyography shows a diminishing amplitutude with repetitive stimulation

    oral anticholinstrase meds such as pyridostigmine or neostigmine are the best initial therapy for myasthenia gravis.
    These meds increase the amount of ACH at the neuromuscular junction b inhibiting its metabolism.
    overdosage of pyridostigmine can lead to diarrhea, cramping, salivation and bronchorrhea.
    if pyridostigmine is ineffective, a patient should have the thymus removed, Thymetcomy is generally only effectve for those under 60 years old. Older pts or those in whom thymetcomy is ineffective, can be treated with prednison, then switched to azathioprine, mycophenolate or cyclosporine. these medications are steriod sparing, acute, severe mysthenic crisis is treated with plasmapheresis or IV immunoglobulin.
    ___________________________

    still pharm but now moved on to oncology meds.
    ________________________

    Imatinib (Gleevec)
    is used for CML chronic myelogenous Leukemia. when the Q is of best initial therapy. Pts will have high white cell count consisteing predominantly of neutrophils of normal appearance. the leuckocyte alakaline phosphatase score should be low and the philadelphia chromosome present.

    Imatinib inhibits production of the philadelphia chromsome. which codes for Bcr?Abl gene, which produces tyrosine kinase.

    Imatinib has very few adverse side effects. occasionally muscle cramps, gynecomastia.
    ________________________________________

    Aromatase inhibitors
    Letrozole
    Anastrazole

    these are the newest adjuvant hormonal therapy after resection of breast cancer. The benefit is best for post menopausal women with estrogen receptor positive breast cancer. It is not clear if aromatase inhibitors anastrazole, letrozole are superior to tamoxifen in terms of efficacy. Adverse effects are certainly less than those of tamoxifen.

    Aromatase inhibitors block the production of estrogen in all tissues, especially in the ovary and adipose tissue, they decrease estrogen levels 97-99%
    the most common side effects are bone loss secondary to inhibition of estrogen, stroke, endometrial cancer, venous thromembolic events, hot flashes and vaginal bleeding, are all less common with anastrazole than tamoxifen. Bone fractures and musculoskeletal pain is more frequent..
    ___________________________________________

    will finish diagnostics after break fast insha'Allah

    __________________________________________________ _
    posst iftar.. ahhhhhhhhhh

    alright diagnostics continuing on with GI

    Bernstein test

    done to diagnose GERD
    it involves the placement of a nasogastric tube into the esophagus with the tip at the end of the esophagus near the gastroesophageal junction. Dilute HCL acid is instilled or dripped into the esophagus in an attempt to reprodice the symptoms of the reflux disease. This is an older test and has been made obsolete by the use of empiric trials of PPIs or the 24 Hr PH monitor.
    just be aware of it but it is for practical purposes replaced.
    ____________________________
    Bleeding scan
    or technetium labeled RBC scanning is performed in pts with small amount of active lower GI bleeding in which the bleeding site could not be identified with endoscopy. Bleeding scan is sensitive but not specific, if the bleeding scan is positive you can't identify the precise etiology of the bleeding. Angiography is another diagnostic option for the patient with persistent, severe GI bleeding without an identified source.

    done on a pt with severe GI bleeding in whom the lower endoscoy is unable to find a source, the bleeding continues and the pt may still require transfusions.
    ________________________________________
    Ceruloplasmin

    serum ceruloplasmin levels and the presence of kayser Fleischer rings on exams are the best initial test for Wislons's dz.
    look for pt with the combination of unexplained hepatic and psychiatric abnormalities with movement disorder.
    An extremely low serum ceruloplasmin levels should be taken as strong evidence for diagnosis of wilson's disease.
    twenty four hour urine copper excretion will be abnormally high. The most accurate test is the lover biopsy. Wislon's disease is a disease of copper overload. This is because of the inability of the lover to incorporate copper into the protein ceruliplasmin for excretion into the bile. This leads to hepatic copper accumulation and injury. This eventually spills into the blood stream and deposits into the brain (behavior and movement changes) kidney (renal insufficiency and corenea (kayser_Fleischer rings)
    ________________________________________________

    Colonoscopy
    recommended as screening for colon cancer.
    General population
    every 10 yrs for pts over 50, colonoscopy is superior to occult blood testing or sigmoidscopy
    single family member with colon cancer
    start at age 40 or 10 years earlier than the family member's diagnosis which ever is earlier.
    Heriditary non-polyposis cancer syndrome (HNPCC or Lynch syndrome)
    Three family members, two generations, one premature. Premature means the cancer was before age 50. Start at age 25 and repeat every 1-2 yrs.
    __________________________________________________ _
    D-Xylose test
    used in determining the etiology of different malabsorption syndromes. D-Xylose is used to differentiate between intestinal malabsortpion and chronic pancreatitis.
    Celiac disease is the most common cause of intestinal mal-absorption. D-Xylose is a sugar that should be absorbed if the small bowel mucosa is intact. It doesn't requie digestive pancreatic enzymes. A low D-Xylose level in the urine indicated that the intestinal mucosa is defective and that D-Xylose was not absorbed
    Lok for a patient with foul smelling stool without blood, who has weight loss, malnutrition, fat in still and positive sudan black stain.
    The most accurate test of course if a biopsy for histology and architecture.
    D-Xylose is abnromal with celiac, tropical sprue and whipple's disease.
    __________________________________________________ _

    Endoscopic retrograde Cholangiopancreatography (ERCP)

    is a direct visualization of the billiary tract.
    ERCP is the most accurate diagnostic test for stones in the billiary system, ERCP is also the most accurate test for primary sclerosing cholangitis. (PSC)

    in sclerosing cholangitis there are multiple intrahepatic and or extrahepatic structures of the billiary tract on cholangiography that give the billiary radicles a beaded appearance.
    The strongest indication for ERCP is whe the pt has high fever, chills, right upper quadrant abdominal pain and jaundice. ERCP i both diagnostic and therapeutic because it can remove the stone. Fpr Primary sclerosing cholangitis look for a pt with history of ulcerative colitis, complaining of pruritus and jaundice with negative hepatitis profile and negative abdominal sonogram.
    __________________________________________________ ____

    esophageal Manometry
    is used to establish the diagnosis of dysphagia or unexplained chest pain where there is no mechanical obstruction.
    esophageal manometry consists of a transducer placed in the esophagus to record pressure.
    Look for dysphaga with inconclusive barium radiograph or upper endoscopy, or barium prior to ding manometry; for example achalasia, diffuse esophageal spasm, or nutcracker esophagus.
    _____________________________________________

    Fecal occult blood testing

    (FOBT) is the screening of normal brown stool for microscopic amounts of blood not grossly visible. Fross blood or dark stool such as melena doesn't occur until >100ml of blood has been lost in the bowel. FOBT will detect as little as 5-10ml of blood
    FOBT is screening test for occult blood in the stool. FOBT is an option for screening for colon ca, in those above the age of 50. In colon ca screening the three methods of screening are
    colonoscopy every 10 yrs
    sigmoidscopy every 3-5 yrs
    FOBT after age of 50
    a positive FOBT must be investigated with a colonoscopy.
    ___________________________________

    Fecal leukocytes

    used to assess the etiology of acute dirrhea
    invasive and inflammatory dirrhea can give blood as well as fecal leukocytes. Diseases such as salmonell, shiegella, campylobacter, etc can give both blood as well as white cells in the stool.
    look for pt with dirrhea with no blood in whom you are tryin to see is there is an invasive form of dirrhea. Fecal leuckocytes are not found with dirrhea secondary to viruses, most protozoans or pre-formed toxins as Bacillus cereus or staph.
    __________________________________________________ ____
    Gastrin Level

    serum gastrin should be measured wen you suspect Zollinger Ellison syndromes
    look for ulcers tat are large >1 cm, recurrent, distal, and multiple. Gastrin levels greater than 1000pg/ml is diagnostic.
    Remember that everyone on PPI has high gastrin level.
    The secretin stimualtion test can confirm ZES when serum gastrin concentration testing is nondiagnostic. In ZES gastrin levels go down with secretin
    ZES is associated with MEN I
    Hyperparthyroidism
    pituitary adenomas
    pancreatic islet cell/Gastrointestinal adenomas.
    ________________________________________________
    Last edited by جوري; 09-13-2008 at 12:24 AM.
    The Medical student Review

    Text without context is pretext
    If your opponent is of choleric temperament, seek to irritate him 44845203 1 - The Medical student Review


  15. #12
    جوري's Avatar Full Member
    brightness_1
    Soldier Through It!
    star_rate star_rate star_rate star_rate star_rate star_rate star_rate star_rate star_rate star_rate star_rate
    Join Date
    Jul 2006
    Location
    من ارض الكنانة
    Gender
    Female
    Religion
    Islam
    Posts
    27,759
    Threads
    1260
    Rep Power
    259
    Rep Ratio
    89
    Likes Ratio
    23

    Re: The Medical student Review

    Today's 5/5
    still on oncology in Pharm.
    _________________
    Riuximab
    is the best therapy to administer in addition to cyclophospamide, doxorubicin, vincristine, prednsione, CHOP as chemotherapy when the patient has non-hodgkin's lymphoma. Rituximab is also used to treat Rheumatoid arthritis when the disease is described as severe and not responsive to other disease modifying drugs. such as methotrexate or tumor necrosis factor inhibitors such as infliximab.

    Rituximab is an antibody that depeltes T-cells. It bunds to the CD20 antigen and prevents cell activation and diferentiation.

    adverse: asscoaited with the development of pneumonia and septic arthritis. IV infusion can cause surface urtricaria, angioedema and bronchospasm.
    _____________________________________
    All -trans Retinoic acid (ATRA)
    is the best initial therapy for acute promyelocytic leukemia (M3) ATRA is added to the usual cytotoxic agents daunorubicin and cytarabine.

    ATRA works by accelerating the terminal differentiation of malignant promyelocytes to mature neutrophils. This leads to apoptosis and complete remission without myelosuppression and bone marrow hypoplasia.
    25% of pts develop retinoic acid syndromes' which consists of fever, edema, pulomnary infiltrates and hypoxia that may be from capillary leak. Hyperleukocytosis with extremely high white blood cell count may occur, it is treated with dexamethasone.
    ________________________________________

    Trastuzumab

    is an additional adjuvant therapy in case of metastatic Br. Ca.. it is used in addition to surgery and chemo.

    it is a monoclonal antibody against the HER2/neu(ErbB-2) member of the epidermal growth factor famiy of cellular receptors. Those are exposed in increased amounts in some patients with breast cancer.

    Trastuzumab is relatively benign when compared to cytotoxic chemotherapy. There is no bone marrow supression and no cardiac toxicity. its side effects are akin to those of other monoclonal antibodies and ma include fever, rash, nausea, dyspnea and rashes. allergic reaction may also be observed.
    _______________________________________
    Anti-thymocyte globulin (ATG)
    is the treatment for aplastic anemia when the question describes panctopenia of unclear etiology and bone marrow transplant is not possible then the answer is ATG. ATG is used to treat aplastic anemia in combination with cyclosporine for those unable to undergo transplanation. ATG can also be used to prevent graft vs host disease and to prevent renal transplant rejection.
    ATG works by removing cytotoxic T-lymphocytes from circulation this allows the bone marrow to regrow.
    ATG can cause serum sickness, rash, anaphylaxix, thrombocytopenia and myalgias.
    _______________________________________________
    Azacytadine

    this is the treatment for myelodsplastic syndrome
    older pt with anemia elevated mean corpuscular volume (MCV) and a small percentage of blasts. The B12 and folate levels will be normal. the bone marrow hypercellular, spenomegaly and hepatomegaly are common.
    azacytadine works by incorporating itself into DNA and RNA and inhibiting methylation of DNA. It is a pyrimidine nucleoside analog of cytadine. It is direcly cytotoxic t abnromal bone marrow hematopoietic cells.

    major toxicities include myelosuppression and rater severe nausea and vomiting.
    _____________________________________________

    part two diagnostics after tarawee7 insha'Allah
    __________________________________________

    ok back with Gi
    __ _________
    Gastrograffin
    water-soluable form of a radio-opaque contrast material is when where there is the possibility that there has been organ perforation and the material may leak through into the peritoneum. Gastrograffin is used as the diagnostic test of choice for Boerhaave's syndrome or esophageal perforation, because barium going through a perforation causes inflammation.
    Look for a patient presenting with a recent history of severe retching followed by severe chest pain, dyspenea, shock, and possible crepitation. Initial chst x-ray would show free air in the mediastinum or pertoneum
    most accurate test however i a ct scan
    _____________________________________
    HFE gene and C282Y mutation both lead to the over absorption of iron in the dueodenum. They are confirmatory tests for hemochromatosis. These genetic tests and an MRI can eliminate the need for liver biopsy to diagnose hemochromatosis.

    HFE gene and C282Y mutation are found in pts wit characteristics of hemochromatosis and abnormal iron studies consistent with hemochromatosis. The patient will have liver disease, skin hyperpigmentation, joint pain, diabetes, infertility, the iron and ferritin levels will be elevated and the iron binding capacity will be markedly diminished.
    ______________________________________________

    HIDA scan
    is used when you suspect acute cholecystitis with a nondiagnostic ultrasonography.
    Answer HIDA when you see either
    Gallstones without gallbladder edema and ultrasound Murphy's sign or
    Gallbladder edema and ultrasound Murphy's sign without gallstones.

    HIDA is taken up by hepatocytes and excreted into bile concentrated in the gallbladder. If the Gallbladder does not opacify, it means that there is an obstruction of the cystic duct. leakage into the pericholecystic space suggests perforation. On a normal scan the gall bladder should opacify.
    The HIDA is not recommended in critically ill pts who should be taken to the operating toom. If the diagnosis is obvious, the HIDA scan is not necessary, surgery is necessary.
    __________________________________________________ ___________

    PAS staining

    PAS staining on a small bowel biopsy is the most accurate test for Whipple's disease/
    Look for chronic multisystem illness, initially rheumatological (80%) then with fat malabsorption (75%), fever (50%) and ocular and neurological complaints.

    initial course of ceftraiaxone followed by trimethorprim sulfamethozazole for one year.
    __________________________________________________ ________________

    24 PH monitoring, is indicated to document abnormal esophageal acid exposure.
    it is appropriate for the following
    Persistent sx of GERD after at least 4 weeks of RX with a PPI
    to excluse GERD in pts with chest pain
    GERD sx with negative upper endoscoy when you are considering antireflux surgical repair.
    to assess the efficacy of antureflux surgery
    to evaluate pts with laryngitis, pharyngitis, chronic cough, non allergic asthma (reflux induced asthma)
    ______________________________
    Last edited by جوري; 09-14-2008 at 01:18 AM.
    The Medical student Review

    Text without context is pretext
    If your opponent is of choleric temperament, seek to irritate him 44845203 1 - The Medical student Review


  16. #13
    جوري's Avatar Full Member
    brightness_1
    Soldier Through It!
    star_rate star_rate star_rate star_rate star_rate star_rate star_rate star_rate star_rate star_rate star_rate
    Join Date
    Jul 2006
    Location
    من ارض الكنانة
    Gender
    Female
    Religion
    Islam
    Posts
    27,759
    Threads
    1260
    Rep Power
    259
    Rep Ratio
    89
    Likes Ratio
    23

    Re: The Medical student Review

    ok.. today 7:56 PM 9/14/2008-- 5/5 still on with Oncology in pharm -- pls again bear in mind this is only the latest and few excerpts of high yield review and by no means is it a substitute for any course you are taking

    Cyclophosamide

    this is an alkylating agent, that is widely used as an immunosuppressive and cancer chemotherapeutic agent.
    it works by cross linking DNA this 'breaks' the DNA and broken DNA can't participate in cell division. Instead, it is metabolized to acrolein.

    cyclophosphamide can cause hemorrhagic cystitis from the acrolein
    it is used for the following conditions:

    Acute leukemia and lymphoma, retinoblastima and a wide variety of cancers
    Wegner's granulomatosis
    polyarteritis nodosa
    lupus nephritis
    nephrotic syndrome (particularly when it does not respond to steroids)
    rheumatoid arthritis that is very severe.
    _________________________________________
    cyclosporine
    cyclosporine is a T-cell inhibiting immunosuppressive agent that is used to prevent organ transplant rejection and to control autoimmune diseases not reponsive to another initial therapy.
    cyclosporine inhibits calcineurin. calcineurin produces interleukin 2. interleukin 2 ativates T cells. Tacrolimus is the other calcineurin inhibitor.
    The most common side effect of cyclosporin is nephrotoxicity. there is increased susceptibility to fungal and viral infections. consumption of grapefruit juice increases its level with potentially adverse effects. Grapefruit juice inhibits the hepatic p450 system
    cyclosporin is the best initial therapy for the following conditions
    organ transplant rejection
    aplastic anemia
    to prevent graft vs host disease
    it is occasionally used as a second line therapy to treat rheumatoidarthritis, ulcerative colitis, Behçet's disease, and psoriasis. It is used to treat myasthenia gravis to spare the pt from chronic steroid use.
    _________________________________________________
    Thalidomide
    is an effective treatment for multiple myeloma (this was taken off the market for extreme tertogenicity way back when, it is back again)
    Thalidominde in MM is equal in efficacy to combination chemotherapy
    other uses are
    Leprosy skin lesions
    apthous ulcerations
    Behçet's disease
    Thalidomide has anti-inflammatory effects that are mediated through its inhibition of tumor necrosis factor, it also inhibits angiogensis.
    Thalidomide is extremely teratogenic, periperal neuropathy and sedation also occur. Dry mouth and skin are common.
    __________________________________________________

    cladribine and pentostatin

    cladribine and pentastatin are the treatment for hairy cell leukemia if the question describes a pt in his or her 50's with pancytopenia, splenomegaly and a 'dry' tap of the bone marrow, then the answer is cladribine or pentostatin. in addition cladribine may have some efficacy in treating lymphma, cheonic lymphocytic leukemia (CLL) and waldernstrom's macroglobulinemia
    cladribine and pentostain are purine analogs tha are naturally toxic to lymphocytes in both the resting and dividing cells.
    Myelosuppresions is the most common adverse effect.
    _______________________________
    67 year old gent evaluated for markedly elevated WBC count of 85,000 of which 90% are normal appearing lymphocytes 'smudge cells' are visible on smear. he has anemia, thrombocytopenia, and an enlared spleen.

    fludarbine is the best therapy for advanced chronic lymphocytic leukemia (CLL) the presence of anemia. thrombocytopena and splenomegaly indicate tha this is advanced stage disease. fludarabine can be combined with cyclophosamide or rituximan. chlorambucil is used only as palliative to decrease the white cell burden of older pts.

    fludarabine works by interfering with DNA synthesis and DNA repair through inhibition of DNA polymerase. The triphosphate form can also be directly incorporated into DNA, resulting in inhibition of DNA synthesis and function. Chlorambucil is a bifunctional alkalyating agent that is rapidly and comletely absorbed from the GI tract. its major metabolite, phenyl acetic acid mustard, is phamarcologically active against CLL
    both fludarabine and chlorambucil cause myelosuppression.
    __________________________________________________ ____

    Now diagnostics
    _______________________

    secretin stimulation test

    this test is rarely used, but it tests for both Zollinger Ellison Syndrom and chronic pancreatitis.
    for ZES you inject secretin IV and measure the level of gastrin in the blood. secretin should inhibit gastrin and lower levels of gastrin in a normal person. in ZES there is a failure to suppress secretin levels

    chronic pancreatitis: normally secretin should induce a large release of bicarbonate from the pancreas into the duodenum. During the test you place a nasogastric tube into the duodenum and measure the release of bicarbonate from the sphincter of oddi. with chronic pancreatitis the pancreas is too burnt out and tired to do anything.

    this test is the answer when a pt has large recurrent and mulciple ulcers, but has a nondiagnostic fasting serum gastrin concentration. for chronic pancreatitis, look for a pt with fat malabsorption with no evidence of celiac disease and a normal D-Xylose test.
    _____________________________________________

    screening upper endoscopy
    should be perfomed in pts with GERD in whom the symtoms have been present for more than 5 yrs. you are screening to see if there is Barrett's esophagus. Which is whn the lower esophagus is transforming from squamous to columnar epithelium. screening for the presence of esophageal varices should be done in those with cirrhosis

    Barrett's esophagus is treated with PPI with a repeat upper endoscopy every 2-3 yrs. Low grade esophageal dysplasia is treated with PPIs with repeat upper endoscopy every 6 months. those with varices should receive propanolol to decrease the risk of bleeding.
    _________________________________________

    72 hr fecal fat
    this is the oldest test for intestinal mal-absorption, as the name implies, you are to collect 3 days worth of stool in order to have a very sensitive test for fat malabsorption. a single test of a sudan black for the presence of fat is usually sufficnet

    72 hour fecal fat colletion is the single most accurate test for malabosprtion; however it is virtually never done.
    ________________________________________

    string test

    this is an old test for giardiasis, it samples duodenal mucosa without the need for endoscopy, a string is swallowed and left in place with one end in the duodenum the other hanging outside the pt's mouth.
    this test is hardly done, stool ova and parasite testing is sufficiently sensitive and less invasive. stool giardia antigen testing by ELISA is the single most accurate test for giardia and considerably less invasive.
    _______________________________________________

    sudan black stain

    stool staining with sudan black is uded to detect steatorrhea as a sign of malabsorption.
    72 hour quantative estimation of stool fat is the most accurate test because of increased sensitivity.
    this is done on pts with chronic pale, greasy, malodorous diarrhea and no evidence of infection

    statorrhea >>intraluminal>>>problem= impaired digestion of nutrients<<<pancreatic exocrine insufficiency>>>or bacterial overgrowth>>>orcholesostatic liver disease


    statorrhea >>mucosal malabsorption>>>>problem=small intestine>>>celiac sprue>>>ortropical sprue>>>or whipple's disease>>>o rintestinal resection..


    statorrhea >>postmucosal>>>problem=transport>>>>intestinal lymphagiectasia>>>or whipple's disease>>>or lymphoma

    _____________________________
    The Medical student Review

    Text without context is pretext
    If your opponent is of choleric temperament, seek to irritate him 44845203 1 - The Medical student Review


  17. #14
    جوري's Avatar Full Member
    brightness_1
    Soldier Through It!
    star_rate star_rate star_rate star_rate star_rate star_rate star_rate star_rate star_rate star_rate star_rate
    Join Date
    Jul 2006
    Location
    من ارض الكنانة
    Gender
    Female
    Religion
    Islam
    Posts
    27,759
    Threads
    1260
    Rep Power
    259
    Rep Ratio
    89
    Likes Ratio
    23

    Re: The Medical student Review

    today's 5/5 finishing off oncology in pharm-- and continue on with GI in diagnostics and move on to pulm
    ______________________

    Flutamide
    Bicalutamide
    Nilutamide

    These are nonsteroidal antiandrogens that are used for prostate cancer and hair loss, they compete with testosterone for binding androgen receptors.

    They can cause gynecompastia, galactorhea, hot flashes, impotence and hepatic injury

    They are the best initial therapy for metastatic prostate ca. as well as hair loss and hirsuitism in women. They should be started before gonadotropin-releasing hormone )HnRH) agonists in metastatic prostate cancer to prevent a flare-up of disease from the initial bump up in testosterone levels. GnRH agonsts initially cause a bump in gonadotropin levels.

    _________________________________________

    47 year old woman with breast ca. presents with back pain, she has spinal tenderness, hyperreflexia of the lower extremities and decreased sensation below her umbilicus extending through the lower half of her body

    steroid adminstration is the most important action in a pt with acute cord compression. This pt has back pain, tenderness, and hyperreflexia. the most important thing is to relieve the pressure on her spinal cord with steroids. Steroids can be given faster and act faster than radiation. steroids decreases the edema surrounding the tumor. MRI is the most accurate test of the compression, but it isn't as important as steroids. Tamoxifen and the aromatase inhibitors, such as anastrazole and letrozle are useful but they don't work acutely.
    ___________________________________________

    68 year old man presents with back pain from metastatic prostate ca. The MRI shows large spinal lesions that are near the cord but not compressing it .

    prostate cancer that is close to impinging on the spinal cord should first be treated with androgen blockade with flutamide, bicalutamide, or nilutamide. Gonadotropin releasing hormone (GnRG) agonists should not be used as first in the case of impending cord compression. because they will cause a brief flare up in luteinizing hormone (LH) and follicale stimulating hormone (FSH) level and subsequently in testosterone levels, perhaps enough to cause cord compression. Block the androgen receptor first. Orchiectomy is the fastst way to lower androgen levels but isn't easy, or as acceptable as medications. Ketoconazole provides near orchiectomy levels of andogens decrease but has much more liver toxicity.
    ____________________________________________

    75 year old man has recently undergone chemotherapy with cisplatin containing regimen, and has developed severe nausea and vomiting
    the most effective therapy for severe nausea and vomiting induced chemotherapy is ondansetron, granisetron, dolasetron, palonosetron or aprepitant, corticosteroids, such as dexamethasone, also inhibit nausea

    ondansetron and others of this group work by selectively inhibiting 5-hydrohytryptamine (5HT3) receptors. Sertonin is released by the small bowel in response to chemotherapy. 5 HT antagonists inhibit stimulation of the vagus nerve that leads to the vomiting reflex. Aprepitant is an inhibitor of the neurokinin-1 receptor.
    _____________________________________________

    37 year old woman comes to see you for evaluation for breast cancer. Her mother and sister both had breast cancer before the age of 45. Her mammograph is normal
    Tamoxifen should be recommended for patients at high risk of breast cancer. Pts. with two or more first degree relatives (mother/sister) with breast cancer have a markedly increased risk of breast cancer--ssomewhere four to six times higher than the average. Tamoxifen decreases the risk of br. ca. in these pts

    Tamoxifen and raloxifene act as estrogen receptor agonists or anatagonists. depending on the tissue. They are estrogen receptor antagonists on breast tissue. This is the basis of their efficacy in treating br cancer. they act as estrogen receptor agonists on the liver and bone. This is the basis of the beneficial effects of raloxifene on osteoporosis. Tamoxifen is an agonist o the uterus, raloxfene isn't.
    Tamoxifen can lead to thromboembolic disease, endometrial cancer, menstural irregularities, sexual dysfunction and hot flashes.
    ________________________________________________

    diagnostics finish Gi move on to Pulmonary.



    Fasting Transferrin saturation

    serum measurement of fasting transferrin saturation is the best initial test to diagnose hereditary hemochromatosis (HH) as well the screening test for asymptomatic family members . Fasting transferrin saturation = (serum iron/TIBC)
    this is the umbrella to tie a case of liver disease with joint pain, diabetes, cardiomyopathy and skin hyperpigmentation and infertility.
    Liver biopsy is the most accurate test
    the pathogensis is genetically determined abnromal increase in intestinal absorption of iton
    the optimal timing for screening family members is between the ages of 18-30 before organ damage (e.g cirrhosis) has occurred.

    _____________________________________________

    serum assay of trypsinogen
    is used when you suspect chronic pancreatits in a patient with steatorrhea. This is the only test that is specific for steatorrhea pancreatic in origin. Trypsinogen evels are low if the fat mal-absorption is from burned out pancreas.

    this is the answer only after Sudan black stain has confirmed fat malabsorption. Antigliadin and anti-endomysial antibodies are normal. Low trypsinogen levels are specifc but not sensitive for chronic pancreatitis.

    __________________________________________________ _____

    urea breath test
    urea breath testing is the measurement of radiolabeled carbon dioxide after oral ingestin of labeled urea. H. Pylori splits urease. The labeled carbon is in the urea and is released as labeled carbon dioxide

    urea breath testing is best used to confirm the eradication of H pylori immediately after treatment. serology does not rapidly improve after treatment. If an ulcer is found at endoscopy, the test of choice will be biopsy, not the urea breath test.

    this is the answer when a pt with epigastric pain that doesn't improve or that recurs after proton pump inhibitor and antibiotics have been given. There is no routine test of cure of H pylori. you don't do the test if the patient feels better after treatment.
    ________________________________________
    Pulmonary

    cover art - The Medical student Review

    this is a bronchoscopy, smal thin fiberoptic scope through the nose, through the vocal cords, through the carina and into mid sized branching bronchus.
    the most accurate diagnostic test for infections such as Pneumocystis or TB. In addition, bronchoscopy is often the most accurate test for lung cancer or other mass lesion of the lung, such as abscess...this is if they are located centrally and are too deep to be reached by a transthoracic ling biosy.
    the most accurate ofcourse if an open lung biopsy.
    ____________________________________

    v94 p309 fig1 - The Medical student Review

    this abnormality is a right upper love cavitary lesion with some post-obstructive penumonia. the most likely cause is a bacterial lung abscess. although TB or a cavitating lung ca. can also give a lung cavity.

    the most accurate test to do is a lung biopsy. must have tissue to obrain a specific microbioloic diagnosis.

    for lung abscess the pt will be febrile with a cough for the last several weeks, weight loss, and possibly malodorous sputum. Lung abscess is most common in those with an increased risk of aspiration, such as alcoholics and pts after a seizure, stroke or intubation.

    >>>>>>>>>>>>>>>>>>>>>>>>>>>
    The Medical student Review

    Text without context is pretext
    If your opponent is of choleric temperament, seek to irritate him 44845203 1 - The Medical student Review


  18. Report bad ads?
  19. #15
    جوري's Avatar Full Member
    brightness_1
    Soldier Through It!
    star_rate star_rate star_rate star_rate star_rate star_rate star_rate star_rate star_rate star_rate star_rate
    Join Date
    Jul 2006
    Location
    من ارض الكنانة
    Gender
    Female
    Religion
    Islam
    Posts
    27,759
    Threads
    1260
    Rep Power
    259
    Rep Ratio
    89
    Likes Ratio
    23

    Re: The Medical student Review

    6:58 PM 9/16/2008-- few mins before breakfast here, so let's see what I can get down.. still on pulmo with diagnostics and today we start opthalmology in pharm
    ___________
    chest2 - The Medical student Review

    this film shows infiltrate, as well major structures of the ling like fissures,. the most likely etiology of this infection is streptococcus pneumonia, although H flu is also possible

    the most accurate test of any bacterial infection is a gram stain of the sputum and a culture. Culture is always the right answer as most accurate diagnostic.
    Xray is best initial test whenever there is cough and shortness of breath
    __________________________

    InfPcP InfThA48JPG 1 - The Medical student Review

    this xray shows bilateral interstitial infiltrates.
    interstitial infiltrates can be caused by viral pneumonia, pneumocystis, mycoplasma, chlamydia, coxiella, and sometimes legionella
    the most accurate test depends on the etiology:
    Mycoplasma, chlamydia, coxiella-serology (antibody titers)
    Legionella-urine antigen testing, culture on specialized media
    PCP-bronchoalveolar lavage, increased LDH
    __________________________________
    chylo fig1a - The Medical student Review

    chest xray with right pleural effusion
    the most common etiology is congestive heart failure or pneumonia, Any infection can cause an effusion, although bacterial infesction is the most common. Less common causes are truama, cancer and connective tissue diseases such as lupus.

    the most accurate test for pleural effusions is a thoracentesis for fluid analysis. The MRI or CT is never accurate for an infectious of neoplastic process. A high protein level indicated infection or cancer. A high cell count an/or low ph (<7.2) indicates an infection.
    ___________________________________________
    pneumothorax 1 - The Medical student Review

    the xray shows a pneumothorax
    pt will present with acute shortness of breath and decreased breath sounds on one side, hyperresonance to percussion and tracheal deviation. It maybe spontaneous, from a central venous cacheter placed in the subclavian vein, or from ruptured bullae from pneumocystis or COPD.
    the next thing is to re-inflate the lung, is there is a 'tension' pneumothorax, then a needle decompression should be performed. All large pneumothoraces require the placement of a chest tube.
    _____________________________________________
    flow volume 1 - The Medical student Review

    flow volume loops are graphical representation of inhalation and exhalation. the inhalation starts at the far right and the tracing moves down. Exhalation begins at the far left and the tracing moves up as you exhale. the loop tells not only how much you inhale and exhale, but also how fast.

    this test is to assess various forms of obstructive and restrictive lung disease; for example to tell the difference between COPD an interstitial lung disease. The flow-volume loop has no use in assessing infections, mass lesions, or reactive airways disease.
    The flow volume loop is a graphical representation meant to uncover subtle defects in vntilation not easily represented on the numerical values of the other parts of the PFT's.

    _________________________________________________
    now ophthalmology/ psychiatry in pharm.
    ____________________________________-

    70 year old woman comes to ER with sudden onset of painful, red eye. The cornea is hazy and edematous. The pupil is fixed at the midpoint and is nonreactive. visual acuity is decreased...
    acute closed-angle glaucoma is treated with medications to increase drainage of aqueous humor and to decrease the production of aqueous humor. Timolol and pilocarpine drops are the best initial therapy. They constrict the pupil.
    Mechanism:
    beta blockers (timolol, betaxolol) constrict the pupils and increase drainage of fluid as well as inflow.
    Cholinergic agonists (pilocarpine) increase drainage of the aqueous humor by opening the canal of schlemm
    Carbonic anyhdrase inhibitors (acetazolamide, dorzolamide) decrease the production of aqueuous humor.
    Alpha adrenerigc agonists (apraclonidine, brimonidine) increase drainage
    prostaglandin analogs (latanoprost or trvoprost) increase drainage.
    if medical therapy doesn't control ocular pressure, a laser trabeculoplasty or iridotomy is performed. this puts a physical hole in the iris allowing for drainage of fluid.
    __________________________________________________ _______

    two pts present with progressive loss of vision. Both are older and generally healthy, without diabetes of HTN. One has atrophic or 'dry' macular degeneration with multiple drusen visible. The other has 'wet' or proliferative disease.

    dry or atrophic, macular degeneration has no proven effective therapy. Laser therapy doesn't help. and the disease will progress slowely over time. Antioxidants, such as vit A,C, and E as well as Zinc may offer some benefits and should be used

    wet (proliferative) exudative macular degeneration has more therapeutic options. Treatment is with vascular endothelial growth factor (VEGF) inhibitors such as ranibizumab, vevacizumab or pegaptanib. These deugs are unique in that they can actually restore vision, not just delay worsening. Laser photocoagulation can be useful; verteporfin is a photosensitizing dye that is injected and actvated by subsequent laser irradiation to produce vascular damage.
    _____________________________________________

    psych.
    Resperidone

    resp. is the rx for chronic schizophrenia.
    the precise mechanism is unknown. Resperidone has an affinity for antagonizing both sertonin and dopamine receptors.

    resp. can cause weakness (asthenia), sedation, difficulty concentrating, and hypotension. It can cause elevated prolactin levels and weight gain, sexual dysfunction. The incidence of tardive dyskinesia is greater with risperidone than with the other second generation antipsychotic medications.
    ________________________________

    olanzapine, ziprasidone, quetiapine, aripiprazole, clozapine, and Risperidone
    are all second generation 'atypical' antipsychotic medications. Clozpine the most efficacious but has the highest incidence of adverse effects such as agranulocytosis.
    the precise mechanism of action is unknown, they all bind to sertonin and dopamine receptors. most also bind to central alpha receptors as well as the histamine receptors.

    they can all cause hypotension, and somnolence, most cause weight gain, Ziprasidone can prolong QT interval the most. Rispridone has the highest incident of tardive dyskinesia and prolactin elevation. Clozapine is the only one that suppresses the bone marrow.
    ______________________________________________

    Selective sertonin reuptake inhibitors
    Fluoxaine
    Paroxtine
    Sertraline
    Fluvoxamine
    Escitalopram

    are the most commonly used anti-depressants
    SSRI"s work by blocking the reuptake of 5-hydroxytryptamine [5-HT] presynaptically. this leaves more 5-HT to stimulate the receptor.
    SSRIs are the best initial therapy for sevre depression. in addition SSRI's treat obsessive compulsive disorder, eating disorders, anxiety, attention deficit disorder and dysmenrrhea.
    there is a possible increased risk of suicide in the first few months after starting medications. SSRI's can also cause sexual dysfunction such as impaired ejaculation.
    >>>>>>>>>>>>>>>>>>>>
    Last edited by جوري; 09-17-2008 at 01:08 AM.
    The Medical student Review

    Text without context is pretext
    If your opponent is of choleric temperament, seek to irritate him 44845203 1 - The Medical student Review


  20. #16
    جوري's Avatar Full Member
    brightness_1
    Soldier Through It!
    star_rate star_rate star_rate star_rate star_rate star_rate star_rate star_rate star_rate star_rate star_rate
    Join Date
    Jul 2006
    Location
    من ارض الكنانة
    Gender
    Female
    Religion
    Islam
    Posts
    27,759
    Threads
    1260
    Rep Power
    259
    Rep Ratio
    89
    Likes Ratio
    23

    Re: The Medical student Review

    today we continue on 5/5
    still on pulmonary in diagnostics and psychiatry in pharm
    ____________________________________

    Spirometry - The Medical student Review

    these are pulmonary function tests. the patient gives a max exhalation after a max inhaltion. that is the FVC, or forced vital capacity. A normal person should be able to exhale at least 80% of the air in one second or FEV1

    in the obstructive pattern as seen on the graph, you see a decrease in the FEV1 and FVC, however FEV1 decreases even more. This gives an abnromally low ration of FEV1 to FVC of <90%. The residual volume goes up and that is why the pt gets a barrel chest and a flat diphragm. This is caused by COPD. in lay man's terms as I struggled with this, ppl in COPD have alot of air trapped that they can't get rid of that is what is called residual volume
    FEV1 - Forced Expiratory Volume in One Second - this is the volume of air which can be forcibly exhaled from the lungs in the first second of a forced expiratory manuever. It is expressed as liters. This PFT value is critically important in the diagnosis of obstructive and restrictive diseases.
    FVC - Forced Vital Capacity - after the patient has taken in the deepest possible breath, this is the volume of air which can be forcibly and maximally exhaled out of the lungs until no more can be expired. FVC is usually expressed in units called liters. This PFT value is critically important in the diagnosis of obstructive and restrictive diseases.
    FEV1/FVC - FEV1 Percent (FEV1%) - This number is the ratio of FEV1 to FVC - it indicates what percentage of the total FVC was expelled from the lungs during the first second of forced exhalation - this number is called FEV1%, %FEV1 or FEV1/FVC ratio. This PFT value is critically important in the diagnosis of obstructive and restrictive diseases.
    ..
    the other graph with the restrictive pattern
    . all the volumes are diminished, but they are diminished proportionately to all the ratios stay normal. restrictive lung disease is caused by interstitial fibrosis.

    the PFT is the most accurate way to distinguish obstructive and restrictive lung dz.
    the most accurate part of PFT is the FEV1/FVC ratio.
    __________________________________

    serum Ace levels
    are associated with sarcoidosis in 75% of pts. Ace levels are contributory to establishing the diagnosis of sarcoidosis --
    Ace is best to do when an african american female presents with cough, dyspnea , chest pain and bilateral hilar adenopathy on chest xray.

    the most accurate test is the presence of noncaseating granulomas on biopsy.
    ____________________________________________

    aerum lpha 1-Antitrypsin level
    (AAT) is measured in a young nonsmoker with emphysema and liver disease, it may also be measured in a pt with a strong fam hx of emphysema, low levels indicate AAT deficiency, in which the body lacks elastase.
    ________________________________

    Arterial blood gas

    directly measures ph, partial pressure of carbon dioxide, and partial pressure of oxygen. The bicarbonate level is determined based on a calculation from the ph and pCO2. This allows you to calculate the alveolar-arterial A-a gradient.. for actualy formula please refer to earlier post.

    the saturation determined from an oximeter is within 1-2% of that obtained on an ABG, so we do not need to do an ABG for O2 saturation. An ABG allows you to know how hard someone is working in order to become oxygenated and where there is life threatening acidosis.
    ABG is the answer f there is a COPD or another possibility of respiratory acidosis. ABG is the answer is there is low serum bicarbonate and there maybe severe metabolic acidosis.
    ____________________________________________
    purified protein derivative PPD

    the PPD is to screen for exposure to TB and the possibility of latent infection. PPD testing is only to screen asymptomatic pts
    the test is positive based on the following
    >or = 5 mm: HIV positive, recent contact with person with active TB, patient with changes on CXR suggestive of prior TB, patients with organ transplants or other immunesuppressed pts,
    >or= 10 mm recent immigrants from countries with high TB prevlance, injection drug users, residents, employees of hospitals, nursing homes, and homeless shelters, and pts with diabetes, renal failure, or hematological malignancies.
    >or = 15mm person with no risk factors for TB
    all pts with +ve PPD should get a CXR to exclude active disease. Positive tests are treated with isoniazid for 9 months.
    Previous vaccinations with BCG has no impact on these recommendations
    a test result >10mm is not to be considered a false positive because of previous BCG.
    __________________________________________________ _

    pharm psychiatry/pulmonary

    32 year old woman comes to ER because of acute onset of facial grimacing, toticollis, oculgyric crisis, and abnormal contractions of the spinal muscles. She has been started on the antipsychotic medication fluphenazine as well metoclopromide for nausea

    the patient has an acute dystonic reaction from both the antipsychotic meds as well as the metocloprimide. her sx consist of an acute oculogyric crisis as well torticollis
    the precise mechanism of acute dystonic rxn is unclear. it occurs because of the antidopaminergic effect of antipsychotic meds as well as metochlopromide.
    Acute dystonic reactions are treated very effectively and rapidly with diphenhydramine and benztropine.
    __________________________________
    lithium
    is an alakali metal that has no effect in a normal person but can treat sx of certain mood disorders. the proposed mechanism may be through suppression of inositol triphosphate. although lithium's precise mechanism is unknown
    adverse effects
    tremor, ataxia, seizures
    nephrogenic diabetes insipidus through inhibition of the ADH receptor.
    hypothyroidism by inhibiting thyroglobulin iodination and coupling
    teratogenesis, particularly ebstein's anamoly
    lithium is indicated for bipolar disorder, and acute mania. it is prophylactic against migraine headache and cluster headaches.
    __________________________

    a pt has been started on anti psychotic meds ziprasidone after being on risperidone at high dose. He becomes febrile to 105 degrees F and is somewhat catatonic with muscle rigidity, confusion and a elevated CPK level.

    neuroleptic malignant syndrome is managed with dopamine agonists, such as bromocriptine. Dantrolene is used as well and can help relieve muscle rigidity
    ________________________________________

    pulmonary

    anticholinergic respiratory therapy
    ipratropium
    tiotropium

    inhaled anticholinergics with ipratropium or tiotropium are the best answer to the question 'what is the best initial drug therapy for a person with chronic obstructive pulmonary disease. They offer additonal bronchodilation for pts with asthma. while they don't work as rapidly as inhaled beta agonists, they can be added to inhaled beta agonists when the pts needs an acute rescue medication for asthma exacerbation. tiotropium has superior efficacy to ipratropium.
    ipratropium/tiotropium antagonizes the effect of ach at muscarinic receptors in the lung. this results in bronchodilation and decrease in the amount of mucus production. they dilate and dry bronchi
    .
    because they are virtually nonabsorbed these drugs almost have no adverse side effects. at high dosages may cause pupil dilation, tachycardia, constipation, dry mouth and urinary retention due to their antichoinergic effects.
    _____________________________________________
    prostacyclin analogs
    Epoprostenol
    Treprostinil
    Iloprost
    Epoprostenol/treprostinol/Iloprost are the correct treatment for severe pulmonary htn. look for pt with slowely progressive SOB that is worst on exertion with a loud P2 heart sound, clear lungs, and pulmonary HTN on echocardiography.

    prostacyclin analogs cause potent pulmonary artery vasodilation, inhibit platelet aggregation and inhibit the proliferation of blood vessles.
    adverse effects are hypotention, flushing, bradycardia, and dizziness.

    >>>>>>>>>>>>>>>>>>>>>>>>>.
    The Medical student Review

    Text without context is pretext
    If your opponent is of choleric temperament, seek to irritate him 44845203 1 - The Medical student Review


  21. #17
    جوري's Avatar Full Member
    brightness_1
    Soldier Through It!
    star_rate star_rate star_rate star_rate star_rate star_rate star_rate star_rate star_rate star_rate star_rate
    Join Date
    Jul 2006
    Location
    من ارض الكنانة
    Gender
    Female
    Religion
    Islam
    Posts
    27,759
    Threads
    1260
    Rep Power
    259
    Rep Ratio
    89
    Likes Ratio
    23

    Re: The Medical student Review

    Today's five and five-- finish pulmonary in diagnostics move on to hematology, and pulmonary in pharm
    ___________________________

    sweat chloride test

    is the gold standard for the diagnosis of cystic fibrosis.
    Pilocarpine is given and the amount of sodium and chloride is measured in sweat.

    this is done for child with chronic respiratory problems such as cough, wheezing and SOB.. there will also be signs of malabsorption such as bulky stools and fat soluble vits deficiency ADEK, also inability to have children, look for an infant with failure to thrive. blood gas will reveal hypoxemia.
    __________________________________

    Heme

    4303018 - The Medical student Review

    this is a hypersegmented neutrophil. Normal neutrophils contain an average of 3.5 lobes per cell. When the average is above 4 lobes, or there are 5% above 5 lobes, or a single 6-lobed cell, hyper segmentation is present. this is the defining feature of megaloblastic anemia.

    hypersegmented neutrophils or megaloblastic anemia is present with vit B12 and folate deficiency.
    the most accurate test is B12 and folate levels. B12 deficiency is also confirmed with an elevated methylmalonic acid level. The etiology of B12 deficiency is confirmed with anti-intrinsic factor and antiparietal cell antibodies, which are diagnostic for pernicious anemia.

    __________________________________________________ _

    T304798A - The Medical student Review
    hairycell nw - The Medical student Review
    both slides show a blood smear, cells with cytoplasmic projections consistent with hairy cell leukemia.
    look for middle aged man presenting with gradual onset of fatigue, splenomegaly is present and maybe massive. The liver is enlarged in 50% of cases. Pancytopenia is the hallmark of hairy cell leukemia.
    the most accurate test is tartarate-resistant acid phosphatase (TRAP) this is done on bone marrow biopsy.
    _____________________________________________

    HeinzBodiesBiteCellsieDegmacytesG6PD 1 - The Medical student Review

    this peripheral smear of RBCs shows Heinz bodies. Heinz bodies are precipitated, oxidized hemoglobin. They are found in glucose-6-phosphate dehydrogenase deficiency (G6PD). G6PD is an x-linked recessive disorder affecting 10-15% of African American males..
    pt is usually healthy until exposed to oxidant stress and suddenly developes intravascular hemolysis resulting in weakness, tachycardia, jaundice and dark urine. The most common type of oxidative stress is from infection, not drugs. Drugs associated wit this stress include sulfa, primiquine, dapsone, quinidine, and nitrofurantoin. Fava beans also cause hemolysis.
    Heinz bodies are removed resulting in cells called bite cells.
    The definitive test for this disorder is a G6PD level. You must wait two months to do the level because the G6PD level will be artificially elevated immediately following a hemolytic event. All the most deficient cells are destroyed.

    __________________________________________

    000p0360 - The Medical student Review
    This is an Auer rod.. which is an eosinophilic needle shaped inclusion in the cytoplasm.
    Auer rods are pathognomonic for acute myelogenous leukemia (AML)
    Auer rods will be found in a pt with pancytopenia, and blasts on peripheral smear of greater than 20% blasts on the bone marrow exam. Histochamical stains demonstrating myeloid enzymes such as peroxidase may further aid in choosing AML as the answer
    _____________________________________-

    Pharm
    pulmo/renal
    ____

    Bosentan

    Bosentan is the RX for severe pulmonary HTN. Look for a pt with severe, progressive shortness of breath and a high pulmonary artery pressure on echocardiogram on right heart catheterization
    Bosentan is a potent inhibitor of endothelin-1. Endothelin-1 causes vasoconstriction of the pulmonary artery as well as proliferation of the smooth muscle artery.
    Bosentan is hepatotoxic and extremely teratogenic.
    __________________________________________________ _

    a young woman has persistent asthma not controlled with albuterol inhaler.

    inhaled steroids have the best efficacy of all the long term asthma controlling medications. Inhaled steroids also have the best effect on mortality.
    Adverse effects and mechanisms:
    Inhaled steroids: oral thrush and dysphonia
    Salmeterol: tremor, heart block, and bradycardia
    Moteleukast: headache
    Cromolyn : stablizes mast cells
    Theophylline: tremors, seizures and arrhythmias. Theophylline has a narrow therapeutic index. it is very toxic with limited efficacy.
    use meds in the following circumstance
    -for Asthmatics not controlled with albuterol inhalers, inhaled steroids have the best efficacy if these do not work long acting beta agonists such as salmuterol should be used.
    Moteleukast is best when the pt. has an atopic allergic disorder such as rhinitis.
    Cromolyn is best when an environmental allergen is at work
    Tehophylline is rarely used.
    ___________________________________________
    Omalizumab
    Omalizumab is the treatment for an asthmatic patient with an extrinsic allergic trigger that isn't controlled by inhaled beta agonosts combined with inhaled steroids or a leukotriene inhibitor, such as monteleukast. Look for a patient with an elevated IgE level or a positive skin test for a specific allergen.
    Omalizumab blocks IgE antibody. This prevents IgE from binding to the mast cell or esoinophil. In this way, it prevents asthma exacernation. Look for a pt who is refractory to inhaled steroids, whom you are trying to keep off oral steroids.
    ___________________________________

    Renal..
    Conivaptan and Tolvaptan
    Conivaptan and tolvaptan are used in the treatment of severe hyponatremia from syndrome of inappropriate antidiuretic hormone secretion (SIADH) a pt with neurological symptoms and a very low sodium level with a normal volume status call for this answer. Conivaptan is used in conjunction with hypertonic saline.
    These medications are vsopressin (Antidiuretic hormone or ADH) receptor antagonists. conivptan and tolvaptan increase free water diuresis and raise the sodium level.
    adverse effects are orthostatic hypotension, peripheral edema, headache, increased thirst, and hypokalemia. serious adverse is A fib.
    _________________________________
    Aliskerin
    Alisekerin is used to rx HTN
    is a direct renin inhibitor. Aliskerin blocks the conversion os angiotensinogen to angiotensin 1. This action leads to a marked decrease in the level of angiotensin II and alodesterone
    Aliskerin can cause hyperkalemia, It doesn't cause cough or angioedema as do ACE inhibitors...

    >>>>>>>>>>>>>>>>>>
    Last edited by جوري; 09-19-2008 at 12:47 AM.
    The Medical student Review

    Text without context is pretext
    If your opponent is of choleric temperament, seek to irritate him 44845203 1 - The Medical student Review


  22. #18
    جوري's Avatar Full Member
    brightness_1
    Soldier Through It!
    star_rate star_rate star_rate star_rate star_rate star_rate star_rate star_rate star_rate star_rate star_rate
    Join Date
    Jul 2006
    Location
    من ارض الكنانة
    Gender
    Female
    Religion
    Islam
    Posts
    27,759
    Threads
    1260
    Rep Power
    259
    Rep Ratio
    89
    Likes Ratio
    23

    Re: The Medical student Review

    5/5

    start with pharm still on renal
    _________________
    a man comes in for a preoperative evaluation of his episodic htn, he has palpitations, headaches, surgical removal of his adrenals is planned

    phenoxybenzamine is the best preoperative medication prior to surgical removal of a pheochromcytoma. this is followed by propanolol.

    phenoxybenzamine is a nonspecific alpha blocker. it is superior to the alpha blocking agents prazosin, terazosin, or doxazosin, propranolol is also a nonspecific beta blocker.

    ____________________________________

    chlorthalidone
    is a thiazide diuretic, inhibits sodium re absorption in the early distal tubule.

    chlorthalidone and thiazide diuretics are the best initial therapy for hypertension, when there are no other compelling indications such as coronoary disease, CHF etc.. chlorthalidone also prevents recurrent kidney stones in calcium overexcreters.
    chlorthalidone causes hypokalemia , hyperglycemia, hyperurucemia and hypercalcemia.
    _____________________________________

    42 year old man comes in for evaluation of hypertension. blood pressure is 146/94
    HTN is defines as blood pressure >140/90. the most effective life-style modification is weight loss. Sodium restriction, dietary modification, and exercise are not as effective as weight loss.

    for specific indication, specific indications, the best therapies are as follows:
    Diabetes: Angiotensin-converting enzymes (Ace) inhibitors, angiotensin receptor blockers ARB's
    coronary artery disease: beta blockers
    CHF, ace inhibitors, beta blockers
    osteoporosis: Thazides they decrease calcium excretion
    prostate hypertrophy, alpha blockers, doxazosin
    depression: don't prescribe beta blockers
    Asthma: don't describe beta blockers.
    ___________________________________________

    Demeclocycline

    demeclocycline is a tetracycline antibiotic that is used in the treatment of chronic syndrome of inapproriate antidiuretic hormone secretion (SIADH)

    demeclocycline inhibits the effect of antidiuretic hormones at the collecting tubule.

    When the question describes a case of SIADH in which the underlying cause cannot be corrected, demecloycline is the answer. It is not used as an antibiotic.
    like all tetracycline antibiotics, demeclocyline can cause photosensitivty.
    _______________________________________________

    Sirolimus, Tacrolimus, and pimerrolimus

    sirolimus, tacrolimus, and pimercrolimus are T-cell inhibiting immuno-suppressive medications used for a number of autoimmune diseases and to prevent organ rejection.

    sirolimus inhibits interleukins. Tacrolimus is a calcineurin inhibitor like cyclocsporine. Pimercellimus' mechanism is not known.
    These medications are used to treat the following conditions:
    sirolimus: prevents renal transplant rejection, prevents coronary stent restenosis. Treats graft versus host disease.
    Tacrolimus: prevents liver transplant rejection and is used to atopic dermatitis, treats graft versus host disease
    pimercolimus is only approved for atopic dermatitis.

    Tarcolimus causes both renal and neurological toxicity. Just as cyclosporine. Bot sirolimus and tacrolimus cause lymphoma..

    >>>>>>>>>>>>>>>>>>>>>>>>>>>

    Diagnostics
    ____
    r CerebSukelMay07Fig1 cont 1 - The Medical student Review

    HeinzBodiesBiteCellsieDegmacytesG6PD 1 - The Medical student Review

    HeinzBodies BiteCellsieDegmacytes G6PD  - The Medical student Review

    these are bite cells. They are formed when Heinz bodies (see previous post) denatured hemoglobin are removed from the cells by the spleen. Macrophages in the spleen remove denatured hemoglobin.

    bite cells are seen with GPD deficiency, which is an x-linked disorder. It is the most common enzymatic disorder of the red blood cells in humans.

    Patients with G6PD deficiency are prone to devloping hemolytic anemia in response to sulfonamides such as dapsone and sulfasalazine. Other precipitating factors are infections, diabetic ketoacidosis, and favism.
    ________________________________________


    schistocytes400 - The Medical student Review
    Schistocytes
    are fragmented red cells and are seen in a variety of shapes and sizes the individual pieces called helmet cells. collectively this is known as intravscular hemolysis or microangipathic hemolytic anemia.
    Fragmented cells are seen in the thrombotic thrombocytopenic purpura (TTP), hemolytic uremic syndrome, major blood grou incompatibility, disseminated intravascular coagulation (DIC), paroxysmal nocturnal hemoglobinuuria, artificial heart valves, and snake bites.
    All forms of hemolysis are associated with elevated LDH, indirect bilirubin, elevated reticulocyt count, and decreased levels of haptoglobin.
    __________________________________________________ _____

    Sid RS - The Medical student Review

    this is ringed sideroblasts. Prussian blue stain is necessary in order to visualize it. The ringed sideroblasts is caused by iron accumulation within mitochondria in the red cell. This is the main findings in sideroblastic anemia...

    look for case of microcytic anemia with an elavted serum iron level in an alcoholic. Acquired sideroblastic anemia can occur as a result of the ingestion of drugs such as alcohol, isoniaizid and chloramphenicol, or toxins such as lead or zinc.
    ringed sideroblasts are also a feature of myelodysplastic syndrome..

    __________________________________________________ __________

    rouleaux nw - The Medical student Review
    Rouleaux formation occurs when red blood cells form stacks or rolls..Rouleaux formation may form due to presence of abnromal globulins or fibrinogen. This formation of red blood cells is found in multiple myeloma and macroglobulinemia. other clues that suggest multiple meyloma might be anemia, hypercalcemia, renal failure and abnormal SPEP or UPEP.
    __________________________________________________ _________

    last and not least.. sob7an Allah.. it has been a long day..

    spherocytes nw - The Medical student Review
    Spherocytes are red blood cells that are almost spherical in shape. They have no area of central pallor like a normal red blood cell...
    the most accurate test for heriditary spherocytosis is the osmotic fragility test. MCHS is not an accurate test..
    the defect is in the ankyrin gene. which leads to spectrin deficiency, which results in membrane instability.
    look for a pt. with recurrent hemolysis, a big spleen, and a family hx of anemia. The CBC reveals anemia and an elevated mean cell hemoglobin concentration (MCHC )

    >>>>>>>>>>>>>>>>>>>>>>>>>
    Last edited by جوري; 09-20-2008 at 05:28 AM.
    The Medical student Review

    Text without context is pretext
    If your opponent is of choleric temperament, seek to irritate him 44845203 1 - The Medical student Review


  23. #19
    جوري's Avatar Full Member
    brightness_1
    Soldier Through It!
    star_rate star_rate star_rate star_rate star_rate star_rate star_rate star_rate star_rate star_rate star_rate
    Join Date
    Jul 2006
    Location
    من ارض الكنانة
    Gender
    Female
    Religion
    Islam
    Posts
    27,759
    Threads
    1260
    Rep Power
    259
    Rep Ratio
    89
    Likes Ratio
    23

    Re: The Medical student Review

    today's 5/5 still in hematology for diagnostics
    __________________________

    ssa 1 - The Medical student Review

    these are sickled cells.

    only sickle cell disease that is homozygous (SS) will produce sickled cells. Homozygous disease (AS) will be 'hematologically' normal

    the most accurate test for sickle cell disease is a hemoglobin electrophoresis.

    ________________________________

    19471 barf6 1 - The Medical student Review this image is really cool, it shows target cells being ingested by macrophages.. anyhow, this is what a target cell looks like

    TargetCells - The Medical student Review

    these cells are associated with liver disease and certain hemoglobinopathies such as sickle cell disease, thalassemia and most notably hemoglobin C disease, iron deficiency can also have target cells.

    the most accurate test if a hemoglobin electrophoresis.

    despite their abrnomal appearance, they don't have a shorter life cycle compared to normal cells.

    _________________________________________

    slide014 tear2 - The Medical student Review

    these are tear drops cells.. and it because they hurt that they are crying .. just checking to see if you are reading lol

    when tear dro cells are either shown or described, you should always think bone marrow disease such as myelofibrosis. look for a case of pancytopenia and massive splenomegaly in which hairy cell leukemia has been excluded. The bone marrow biopsy will show reticulin fibers.

    __________________________________________________

    anti-intrinsic factor antibodies, antiparietal cell antibodies

    are highly confirmatory for pernicious anemia. They are nearly 100% specific for the disease. Pernicious anemia is an autoimmune disease in which you become allergic to you own IF, and gastric parietal cells.

    Answer anti-IF and antiparietal cell antibodies when you see a case of B12 deficiency and you want to determine the etiology. The case will describe a pt in whom B12 is low, or te methylmalonic acid level is high. These antibodies are not to diagnose B12 deficiency, they are to determine the etiology.

    ______________________________________________


    BurrCells - The Medical student Review

    burr cells, also called echinocytes, are RBC's with many blunt, regular sicules. They are present in end stage renal disease and liver disease...

    acanthocytes2001 - The Medical student Review

    compared to spurr cells or acanthocytes, which have only a few irregular spicules, they are also seen in severe liver disease and in some forms of hereditary spherocytosis, nd myelodysplasia.

    ________________________________________________

    ok Now Pharm.. still on Renal

    _____

    Clonidine

    this is an old hypertensive in the archives lol.. no longer recommended for the rx of HTN because of the large number of adverse side effects and limited evidence of mortality benefit

    clonidine is a centrally acting alpha 2 agonist. This decreases the release of vasconstrictive neurotransmitters, such as norepinephrine.

    clonidine can cause sedation, and dry mouth and rapid withdrawl which can lead to hypertensive crisis.

    clonidine has no room in rx of HTN however it is used as an adjunct to treat opaite withdrawl, diarrhea related to diabetes and occasionally tourette's syndrome.

    ________________________________________

    a woman comes to the ER dept. after falling while running a marathon, she is also taking a statin medication. she has dark urine that is dipstick positive for large amounts of blood. There are no red blood cells seen on the microscopic examination. Her serum bicarbonate is low.

    this pt most likely has rhabdomyolysis. The dark urine that is dipstick positive for blood with o red cells seen is myoglobin.

    te best initial therapy for rhbdomyolysis is vigorous hydration, bicarbonate adminstration and possible mannitol.
    the hydration and mannitol decrease the amount of time that the nephrotxic myoglobin is in contact with the kidney tubule. Bicarbnate prevents precipitation of myoglobin in the kidney tubule. In addition, it drives potassium intracellularly by causing an alkalosis.

    The cause of death in rhbdomyolysis is hyperkalemia and acidosis. both of these can cause an arrhythmia.

    __________________________________________________

    a man has just undergone combination chemotherapy for widespread non-hodgkins lymphoma. The EKG shows peaked T waves

    Hyperkalemia with EKG changes should be immediately treated with intravenous calcium chloride or calcium gluconate. This will protect the heart from an arrhythmia. Insulin and glucose should be used to drive potassium intracellularly, but they take 15-20 minuted to work. The calcium will protect the heart from an arrhythma while the Insulin and glucose take time to work.

    Kayexcelate is a sodium/potassium cation-exchane resin and is adminstered orally to remove potassium from the body vua the gastrointenstinal tract. It takes several hours to work.

    Tumor lysis syndrome can be prevented by adminstering intravenous hydration and allopurinol before chemotherapy is given.

    __________________________________________

    Azathioprine
    is a purine inhibitor that is used to control autoimmune diseases.
    azathioprine is converted to 6-mercaptapurine, whose metabolites inhibit purine metabolism. Purine synthesis is needed for the proliferation of B and T lymphocytes. This redices antibiody production.

    Azathiprine can be bone marrow suppressive and can cause infection. Allopurinol interfers with azathiopurine, because the active metaboolite is metabolized by xanthine oxidase. Pancreatitis happens from direct toxicity.

    Azathioprine is used for the following conditions

    Inflammatory bowel disease
    Vasculitis and mysthenia gravis to spare the pt from chronic steriod use
    lupus nephritis that isn't responding to steroids, cyclphasapmide, or mycophenolate
    renal translanation
    rehumatoid arthritis

    _________________________________________

    mycophenolate mofetil
    is an immunosuppressive agent with less toxicity than cyclophosphamide, azathioprine or cyclosporine.

    mycophenolate inhibits purine synthesis, this prevents the proliferation of lymphocytes.
    mycophenolate has a few adverse effects.
    It can cause infection via leukopenia

    mycophenolate is the drug of choice for lupus nephritis. it is superior to cyclophasmide, oter indications are the following
    organ transplant rejection prevention.
    Nephrotic syndrome, not responsive to sterioids and cyclophosamide
    it can be used as a second line treatment for myasthenia gravis, atopic dermatitis, vasculitis and crohn's disease to prevent dteroid dependence

    >>>>>>>>>>>>>>>>>>>>>>>>>>>
    The Medical student Review

    Text without context is pretext
    If your opponent is of choleric temperament, seek to irritate him 44845203 1 - The Medical student Review


  24. Report bad ads?
  25. #20
    جوري's Avatar Full Member
    brightness_1
    Soldier Through It!
    star_rate star_rate star_rate star_rate star_rate star_rate star_rate star_rate star_rate star_rate star_rate
    Join Date
    Jul 2006
    Location
    من ارض الكنانة
    Gender
    Female
    Religion
    Islam
    Posts
    27,759
    Threads
    1260
    Rep Power
    259
    Rep Ratio
    89
    Likes Ratio
    23

    Re: The Medical student Review

    today's 5/5 still on heme in diagnostics, renal in pharm and will move on to rheumatology insha'Allah
    ____________________________________

    Bleeding Time

    a test for the ability of the platelet to adhere to the endothelial lining of the capillary. Do not use it when the platelet count is abnormally low. Everyone with a low platelet count will have an abnormal bleeding time.

    a blood pressure cuff is inflated to 40mm Hg to encourage the blood vessles, then small cuts are made, the test is the amount of time it takes for the bleeding to stop.

    bleeding time is the answer, when the pt seems to have a bleeding disorder, examples are von willebran's disease or uremia induced platelet dysfunction.

    the most accurate test for von willebrand factor level, ristocetin cofactor assy, or in vitro platelet aggregation studies..

    __________________________________________________

    decay-accelerating factor (DAF) or CD55/CD59 assay

    Cd55/59 is a test for paroxysmal nocturnal hemoglobinuria (PNH), also knows as decay accelerating factor. DAF is a membrane component of the complement system.

    Assay for DAF is the most accurate test for diagnosing PNH

    DAF when you see a pt with intermittent dark urine, particularly in the morning, pancytopenia, and large vessel thrombosis..

    _____________________________________________

    Cold Agglutinins

    are IgM autoantibodies directed against RBC antigens. Titers are measured when Cold Agglutinins disease, which is a type of autoimmune hemolytic anemia, is suspected.

    high titres have been found to be associated with malignancies such as lymphoma, CLL, and Waldenstrom's macroglobulinemia, as well as mycoplasma infections and infectious mononucleosis.

    Answer cold agglutinins when you see a patient with hemolytic anemia as well as skin discoloration when exposed to cold temperatures. Do not confuse this with cryoglobulins. which give purpuric skin lesions and glomerulonephritis.
    ________________________________________________

    Coombs' Agglutinin test

    coombs test measures the presence of antibodies against red blood cells.

    Coombs tests is used for the diagnosis of autoimmune hemolysis. A positive coombs tests shows the agglutination of the patient's red blood cells.
    a total of 20-50% of coombs positive hemolytic anemia is iodiopathic. the most common identifies etiologies are PCN, quinidine, alphamethyldopa, sulfa drugs and ASA.

    _____________________________

    D-Dimer/Fibrin split products

    D -dimer is the by=product of the degradation of fibrin by plasmin. Fibrin split products (FSPs) are produced by the activation of fibrinogen by thrombin. They are abnromal only if they are present in increased amounts.
    D-dimer can be measured by levels either latex agglutination (more rapid) or Elisa (more accurate). Elisa is far more sensitve.
    elevated levels indicate pathology involving clot formation and lysis from activation of the coagulation cascade. They provide no information about platelet function.
    Elevated D dimer/FSP levels are evidence of DIC, a negative result from the ELISA rules out DVT and PE but a positive results does not confirm the diagnosis.
    order these tests to confirm suspected DIC and to rule out PE in pts with a low pre=test probability

    ____________________________

    pharm/Renal

    Man comes in with severe headache, confusion, blood pressure of 190/124, head CT is normal
    Hypertensive emergencies are treated with IV agents that allow rapid lowering of BP with greater control han oral agents. Labetolol is the best initial therapy for hypertensive crisis. Labetolol is a mixed alph and nonspecific beta blocker that is very effective. Nitroprusside is used in the intensive care unit and usually requires monitoring with an arterial line. Other choices are enalaprilat, hydralazine or nicardipine.

    in a hypertensive emergency, you should not lower the blood pressure more than 25% over the first several hours.
    blood pressure lowered too rapidly can result in a stroke.

    __________________________________________________ _

    a patient with end stage renal disease is preparing to start dialysis. his phosphate level is markedly elevated ..

    hyperphosphatemia from renal failure is treated with calcium carbonate, calcium acetate, sevelamer or lanthanum . Cinacalcet is a calimimetic agent that treats secondary hyperparathyroidism. it is used is there is hypercalemia from secondary hyperparathyroidism.

    Calcium carbonate, calcium acetate, sevelamer and lanthanum all bind phosphate in the bowel. the calcium containing medications form complex with phosphate, which is then excreted in the sool. Sevelamer is a nonabsorbed polymer that also binds phosphate in the bowel. Cincalcet acts like calcium on the parathyroid gland and inhibits the release of parathyroid hormone.
    All of these medications case some gastrointenstinal discomfort. calcium carbonate and calcium acetate can lead to hypercalcemia. if this happens, treatment should be switched to sevelamer or lanthanum.

    __________________________________________________ ____

    Pharm rheumatology

    Tumor necrosis factos (TNF) inhibitors
    infliximab, etanercept, adalimumab

    tumor necrosis factor inhibitor are used to treat the following conditions:
    inflammatory bowel disease --particularly crohn's with fistula formation

    Rheumatoid arthritis, that is not responsive to methotrexate as a DMARD
    psoriatic arthritis, when it is moderate to severe, as an alternative to systemic therapy such as methotrexate or UV light

    ankylosing spondylitis
    TNF inhibitor are immunosuppressive but less toxic than steroids.
    TNF inhibitors can reactivate or worsen serious bacterial infections by inhibiting the immune system. TB reacivation in PPD positive pts is te most important adverse effect.
    All pts who go on TNF inhibitor should have a PPD test done first. TNF inhibitors are also associated with the development of lymphoma.

    ___________________________________________

    methotrexate..

    this is the answer as to which slows down the progression of disease, in virtually all pts wit rhumatoid arthritis. It is the number one DMAR for rheumatoid arthritis.
    it is also used for the following
    severe psoriasis, particularly psoriatic arthritis
    leukemia, lymphoma and certain solid tumors.

    Methorexate causes
    liver toxicity
    pulmonary fibrosis
    bone marrow suppression (myelosuppression)
    kidney damage (precipitation of methotrexate crystals)

    ______________________________________
    rheumatoid arthritis and alternate DMARDS

    hydroxychloroquine
    sulfasalazine
    anakinra
    abatacept
    leflunomide

    Alternative DMARDS to methotrexate are the correct answer to the following
    pt is intolerant to methotrexate
    methorexate fails to control disease
    cases of mild disease where it is preferable to avoid the toxicity of methotrexate, sulfasalazine ad hydroxycloroquine can be used initially in this way..

    adverse effects are as follows..
    hydroxychloroquine-- renal damage, hemolysis
    sulfasalazine--rash, heptaitis, agranulocytosis
    anakinra--interluekin-1 antagonist-causes neutropenia
    abatacept-- inhibits t cell activation-->infections
    leflunomide-- inhibits pyramidine synthesis--causes rash, alopecia, mylosuppression and liver dysfunction...

    >>>>>>>>>>>>>>..
    The Medical student Review

    Text without context is pretext
    If your opponent is of choleric temperament, seek to irritate him 44845203 1 - The Medical student Review



  26. Hide
Page 1 of 7 1 2 3 ... Last
Hey there! The Medical student Review Looks like you're enjoying the discussion, but you're not signed up for an account.

When you create an account, we remember exactly what you've read, so you always come right back where you left off. You also get notifications, here and via email, whenever new posts are made. And you can like posts and share your thoughts. The Medical student Review
Sign Up

Similar Threads

  1. Replies: 2
    Last Post: 05-26-2010, 02:35 PM
  2. Book Review
    By Beardo in forum Creative Writing & Art
    Replies: 2
    Last Post: 11-13-2009, 03:51 PM
  3. Replies: 1
    Last Post: 08-28-2007, 11:08 PM
  4. Replies: 0
    Last Post: 04-21-2007, 01:10 PM
  5. book review
    By partysoverkids in forum General
    Replies: 1
    Last Post: 02-19-2006, 10:50 PM

Posting Permissions

  • You may not post new threads
  • You may not post replies
  • You may not post attachments
  • You may not edit your posts
  •  
create